Gastrointestinal Tract 2 (7-1 - 14-1)

General

7-1 (100B35) Which of the following statements is true about colon adenoma?

a) It is often found in individuals in their 30s.

b) It frequently occurs in the ascending colon.

c) The larger the adenoma, the higher the risk of cancerous complications.

d) It is often accompanied by mucous stools.

e) Pedunculated adenomas are common.

 

The correct statement is:

c) The larger the adenoma, the higher the risk of cancerous complications.

Explanation:

  • Colon adenomas are benign tumors of the colon that have the potential to develop into colorectal cancer. The risk of transformation into cancer increases with the size of the adenoma, making regular screening and removal important for prevention.

  • Statement (a) is misleading because colon adenomas are more commonly found in individuals over the age of 50, which is why screening recommendations often start at this age.

  • While adenomas can occur throughout the colon, statement (b) does not accurately represent the risk associated with their location.

  • Statement (d) regarding mucous stools is not specifically associated with the presence of colon adenomas.

  • Statement (e) about the prevalence of pedunculated adenomas is too general. Both pedunculated (on a stalk) and sessile (flat) adenomas are found in the colon, but the shape does not inherently dictate the risk of cancer.

7-2 (101F31) Which combinations of colorectal disease and complications are correct? Select two.

a) Intestinal tuberculosis - Hemolytic uremic syndrome

b) Amoebic dysentery - Liver abscess

c) Ulcerative colitis - Primary sclerosing cholangitis

d) Irritable bowel syndrome - Uveitis

e) Colorectal carcinoids - Erythema nodosum

 

The correct answers are:

b) Amoebic dysentery - Liver abscess

c) Ulcerative colitis - Primary sclerosing cholangitis

Explanation:

  • Amoebic dysentery caused by Entamoeba histolytica can lead to liver abscesses when amoebae invade the liver through the portal vein. This is a known complication of amoebic dysentery and represents a direct extension of the infection from the colon.

  • Ulcerative colitis is an inflammatory bowel disease that can have several extraintestinal manifestations, including primary sclerosing cholangitis (PSC). PSC is a chronic liver disease characterized by inflammation and fibrosis of the bile ducts and is more common in people with ulcerative colitis.

  • Intestinal tuberculosis leading to hemolytic uremic syndrome (HUS) is not a commonly recognized association. HUS is typically associated with certain bacterial infections, especially Shiga toxin-producing E. coli, not tuberculosis.

  • Irritable bowel syndrome (IBS) is primarily a functional gastrointestinal disorder and is not typically associated with uveitis, an inflammation of the uvea in the eye.

  • Colorectal carcinoids leading to erythema nodosum is not a standard association. Erythema nodosum is a skin condition that can be associated with various systemic diseases, but it is not specifically linked to colorectal carcinoids.

7-3 (102D11) Which statement is true about pseudomembranous enteritis? a) It is caused by the growth of pathogenic E. coli.

b) It develops with sudden fresh melena.

c) In enema radiography, it is depicted as a barium pooling image.

d) Endoscopic examination is performed for diagnosis.

e) Treatment involves administering broad-spectrum antimicrobials.

 

The correct answer is:

d) Endoscopic examination is performed for diagnosis.

Explanation:

  • Pseudomembranous enteritis, also known as pseudomembranous colitis, is primarily caused by the overgrowth of Clostridioides difficile (C. difficile), not pathogenic E. coli, making option (a) incorrect.

  • The condition is characterized not necessarily by fresh melena but by diarrhea that may contain mucus or blood, making option (b) inaccurate for this context.

  • Enema radiography may show specific changes in the colon, but the description of "barium pooling image" does not specifically pertain to pseudomembranous enteritis, making option (c) misleading.

  • Endoscopic examination is indeed used for diagnosing pseudomembranous enteritis. During the procedure, the characteristic pseudomembranes can be observed on the colonic mucosa, confirming option (d) as correct.

  • The treatment for pseudomembranous enteritis involves targeted antimicrobial therapy against C. difficile, such as metronidazole or vancomycin, rather than broad-spectrum antimicrobials, which can actually precipitate or exacerbate the condition by disrupting normal gut flora. Therefore, option (e) is incorrect.

7-4 (102D19) Which causes bowel obstruction? Select two.

a) Infectious enteritis

b) Colon cancer

c) Sigmoid volvulus

d) Irritable bowel syndrome

e) Rectal prolapse

 

The correct answers are:

b) Colon cancer

c) Sigmoid volvulus

Explanation:

  • Colon cancer can lead to bowel obstruction as the tumor grows large enough to block the passage of digested food and waste. This obstruction can cause symptoms such as abdominal pain, constipation, and bloating.

  • Sigmoid volvulus involves the twisting of the sigmoid colon around its mesenteric axis, which can cause a blockage in the intestine. This condition is more common in elderly patients and can present with symptoms of obstruction, such as abdominal pain, distension, and constipation.

  • Infectious enteritis (a) typically causes diarrhea and is not a common direct cause of bowel obstruction.

  • Irritable bowel syndrome (d) is a functional gastrointestinal disorder characterized by symptoms like abdominal pain, bloating, and altered bowel habits, but it does not cause physical obstruction of the bowel.

  • Rectal prolapse (e) involves the protrusion of the rectal walls through the anal opening but does not directly cause bowel obstruction, although severe cases might indirectly affect bowel movements.

7-5 (102E28) Which is not a factor that determines the stage of colorectal cancer?

a) Histological classification

b) Wall depth

c) Lymph node metastasis

d) Liver metastases

e) Peritoneal metastases

 

The correct answer is:

a) Histological classification

Explanation:

  • Histological classification refers to the microscopic examination of cancer cells to determine their type and characteristics. While important for understanding the type of cancer and guiding treatment decisions, it is not a direct factor in determining the stage of colorectal cancer.

  • Wall depth (b), lymph node metastasis (c), liver metastases (d), and peritoneal metastases (e) are all factors used in the TNM (Tumor, Node, Metastasis) staging system to determine the stage of colorectal cancer. The stage of cancer is critical for determining the prognosis and the most appropriate treatment options.

7-6 (102G34) Which is appropriate in the response to intra-abdominal abscesses?

a) Arterial embolization

b) Drainage

c) Ethanol injection

d) Radiofrequency ablation

e) Sclerotherapy

 

The correct answer is:

b) Drainage

Explanation:

  • Drainage is the primary and most appropriate treatment for intra-abdominal abscesses. It involves the removal of pus from the abscess through surgical or percutaneous methods, helping to resolve the infection and alleviate symptoms.

  • Arterial embolization (a) is used to control bleeding, not typically for abscess treatment.

  • Ethanol injection (c) and radiofrequency ablation (d) are treatments used for certain tumors or cysts, and are not standard treatments for abscesses.

  • Sclerotherapy (e) is used to treat varicose veins and other vascular anomalies, not abscesses.

7-7 (103I22) Which combination is correct? Choose two.

a) Lanz tender point - Appendicitis

b) Murphy's sign - Acute Pancreatitis

c) Blumberg's sign - Peritonitis

d) McBurney's point tenderness - Ischemic Colitis

e) Courvoisier's sign - Fulminant Hepatitis

 

The correct answers are:

a) Lanz tender point - Appendicitis

c) Blumberg's sign - Peritonitis

Explanation:

  • Lanz tender point is associated with appendicitis, characterized by localized tenderness in the right lower quadrant of the abdomen.

  • Blumberg's sign is indicative of peritonitis, where there is rebound tenderness upon quick release of deep pressure in the abdominal area, suggesting irritation of the peritoneum.

  • Murphy's sign is a clinical finding associated with gallbladder inflammation (acute cholecystitis), not acute pancreatitis. It is elicited by asking the patient to take a deep breath while pressing on the right subcostal area, causing pain and halting of inspiration if positive.

  • McBurney's point tenderness specifically indicates appendicitis, not ischemic colitis, and is found one-third of the distance from the anterior superior iliac spine to the umbilicus.

  • Courvoisier's sign, the presence of a palpably enlarged gallbladder which is painless, typically indicates an obstruction of the bile ducts often due to pancreatic or biliary obstruction, not fulminant hepatitis.

7-8 (104D2) Which statement is correct for Hereditary Nonpolyposis Colorectal Cancer (HNPCC)?

a) Poor prognosis.

b) Onset at a young age.

c) It predominantly occurs in the left side of the colon.

d) There is an APC gene abnormality.

e) Lower incidence of cancers in other organs.

 

The correct answer is:

b) Onset at a young age.

Explanation:

  • Hereditary Nonpolyposis Colorectal Cancer (HNPCC), also known as Lynch syndrome, is characterized by an increased risk of developing colorectal cancer and other cancers at a younger age, typically before the age of 50. This is a hallmark feature of HNPCC.

  • HNPCC is not necessarily associated with a poor prognosis (a); prognosis depends on the stage at diagnosis and the specific cancers involved.

  • HNPCC cancers can occur anywhere in the colon (c), but they are not exclusively or predominantly found in the left side of the colon.

  • The APC gene abnormality (d) is associated with Familial Adenomatous Polyposis (FAP), not HNPCC. HNPCC is associated with mutations in DNA mismatch repair (MMR) genes, such as MLH1, MSH2, MSH6, and PMS2.

  • HNPCC is associated with a higher incidence of cancers in other organs, including the endometrium, ovary, stomach, small intestine, hepatobiliary tract, upper urinary tract, brain, and skin (e), contrary to the statement that there is a lower incidence of cancers in other organs.

7-9 (105A3) What are the symptoms of irritable bowel syndrome?

a) Fever

b) Edema

c) Blood in the stool

d) Postprandial abdominal pain

e) Weight loss

 

The correct answer is:

d) Postprandial abdominal pain

Explanation:

  • Irritable bowel syndrome (IBS) is characterized by a group of symptoms that include abdominal pain or discomfort, often reported after eating (postprandial abdominal pain), and changes in bowel habits (such as diarrhea, constipation, or alternating between the two). It is a functional gastrointestinal disorder, meaning symptoms have no identifiable organic cause.

  • Fever (a) and edema (b) are not typical symptoms of IBS. These symptoms are more indicative of other medical conditions.

  • Blood in the stool (c) is not a common symptom of IBS. The presence of blood in the stool suggests other conditions, such as inflammatory bowel disease or colorectal cancer, and warrants further investigation.

  • Weight loss (e) can occur in individuals with IBS, primarily if the disorder affects eating habits or food intake due to fear of triggering symptoms. However, significant weight loss is not a hallmark symptom of IBS and could indicate other health issues.

7-10 (106A9) Which is appropriate in the response to steroid-resistant severe ulcerative colitis?

a) Add treatment for amoebic dysentery.

b) Observe the entire large intestine with enema duplex contrast.

c) Rest the intestinal tract with the administration of morphine.

d) Perform a search for Cytomegalovirus.

e) Administer nonsteroidal anti-inflammatory drugs (NSAIDs).

 

The correct answer is:

d) Perform a search for Cytomegalovirus.

Explanation:

  • In cases of steroid-resistant severe ulcerative colitis, it's crucial to explore underlying causes that could be contributing to the disease's severity or resistance to treatment. One such cause can be a superimposed infection with Cytomegalovirus (CMV), which can exacerbate colitis and make it resistant to conventional therapies. Identifying and treating CMV can improve outcomes.

  • Adding treatment for amoebic dysentery (a) is only appropriate if there's evidence of amoebiasis, which would be unusual in this context without specific epidemiological risk factors or symptoms.

  • Observing the entire large intestine with enema duplex contrast (b) can be useful for diagnostic purposes but does not address the immediate need to manage severe, steroid-resistant disease.

  • Resting the intestinal tract with the administration of morphine (c) might temporarily relieve symptoms but does not address the underlying disease activity or the specific issue of steroid resistance.

  • Administering nonsteroidal anti-inflammatory drugs (NSAIDs) (e) is generally contraindicated in patients with ulcerative colitis, as NSAIDs can exacerbate the condition.

7-11 (107A5) Which of the following patients is not a candidate for treatment of Crohn's disease with biologics (anti-TNF-α antibody preparations)?

a) Patients 65 years of age or older

b) Patients with active tuberculosis

c) Patients with enterocutaneous fistula (external fistula)

d) Patients taking other Crohn's disease medications

e) Patients after induction of remission with biologics

 

The correct answer is:

b) Patients with active tuberculosis

Explanation:

  • Patients with active tuberculosis are not candidates for treatment with anti-TNF-α antibody preparations due to the risk of exacerbating tuberculosis infections. These biologic medications suppress the immune system, potentially allowing latent tuberculosis infections to become active and worsen.

  • Age alone, such as patients 65 years of age or older (a), does not necessarily preclude the use of biologics unless there are specific health concerns that make biologics unsafe for the patient.

  • Patients with enterocutaneous fistula (c) may benefit from biologics, as these drugs can help close fistulas and reduce inflammation.

  • Patients taking other Crohn's disease medications (d) can often use biologics in combination with other treatments, depending on their specific condition and treatment plan.

  • Patients after induction of remission with biologics (e) may continue to use biologics for maintenance therapy to prevent disease relapse.

7-12 (108A16) Which characteristics are associated with Crohn's disease? Choose two.

a) Paving stone appearance

b) Annular ulcer

c) Small bowel stenosis

d) Melanosis coli

e) Continuous lesions

 

The correct answers are:

a) Paving stone appearance

c) Small bowel stenosis

Explanation:

  • Paving stone appearance is a classic endoscopic finding in Crohn's disease, characterized by the cobblestone-like appearance of the intestinal mucosa due to the presence of ulcers and edema.

  • Small bowel stenosis is also a common feature of Crohn's disease, resulting from chronic inflammation, fibrosis, and subsequent narrowing of the small intestine, leading to obstructive symptoms.

  • Annular ulcers (b) are not specifically characteristic of Crohn's disease and can occur in various gastrointestinal disorders.

  • Melanosis coli (d) is a condition typically associated with prolonged laxative use and is unrelated to Crohn's disease.

  • Continuous lesions (e) are more typical of ulcerative colitis, another form of inflammatory bowel disease. In contrast, Crohn's disease is characterized by "skip lesions," where inflamed areas are interspersed with healthy tissue.

7-13 (108E22) Which combination of disease and factor that relieves abdominal pain is correct?

a) Cholelithiasis - Fat intake

b) Chronic pancreatitis - Alcohol consumption

c) Duodenal ulcer - Fasting

d) Generalized peritonitis - Body movement

e) Irritable bowel syndrome - Defecation

 

The correct combination of disease and abdominal pain relief is:

e) Irritable bowel syndrome - Defecation

Explanation:

  • Cholelithiasis (a) is characterized by gallstones and pain often occurs after fat intake, not relieved by it.
  • Chronic pancreatitis (b) can be exacerbated by drinking alcohol, not relieved.
  • Duodenal ulcer (c) pain typically decreases or is relieved by eating (not fasting), which buffers stomach acid.
  • Generalized peritonitis (d) is a severe inflammatory condition where movement typically increases pain, not relieves it.
  • Irritable bowel syndrome (e) is often associated with abdominal discomfort or pain that is relieved by defecation.

7-14 (109I13) Which combination of gastrointestinal disorders and their complications is incorrect?

a) Crohn's disease - anal fistula

b) Pseudomembranous enteritis - liver abscess

c) Meckel's diverticulum - intestinal obstruction

d) Ulcerative colitis - toxic megacolon

e) Ischemic colitis - colon perforation.

 

The incorrect combination is:

b) Pseudomembranous enteritis - liver abscess

Explanation:

  • Pseudomembranous enteritis is typically associated with Clostridium difficile infection in the colon and does not usually cause liver abscesses. Liver abscesses are more commonly related to conditions that affect the liver directly, such as a biliary tract infection.
  • The other pairs listed (a, c, d, e) correctly associate common complications with their respective gastrointestinal disorders. Crohn's disease can lead to anal fistulas, Meckel's diverticulum can cause intestinal obstruction, ulcerative colitis may result in toxic megacolon, and ischemic colitis can lead to colon perforation.

7-15 (109I37) What are two causes of superior mesenteric artery occlusion?

a) Liver cirrhosis

b) Atrial fibrillation

c) Chronic pancreatitis

d) Atherosclerosis

e) Addictive constipation.

 

The two causes of superior mesenteric artery occlusion from the options provided are:

b) Atrial fibrillation

d) Atherosclerosis

Explanation:

  • Atrial fibrillation can lead to the formation of blood clots, which may dislodge and occlude the superior mesenteric artery, disrupting blood flow to the intestines.
  • Atherosclerosis, the buildup of plaques in the arteries, is a common cause of arterial occlusion, including in the superior mesenteric artery, leading to reduced blood flow and potential intestinal ischemia.

7-16 (111I19) Which of the following colorectal diseases can lead to the development of colorectal cancer?

a) Diverticulitis

b) Crohn's disease

c) Ischemic enteritis

d) Megacolon

e) Ulcerative colitis

 

The correct answer is:

e) Ulcerative colitis

Explanation:

  • Ulcerative colitis is a chronic inflammatory bowel disease that affects the lining of the colon and rectum. Long-standing inflammation in these areas increases the risk of developing colorectal cancer. Patients with ulcerative colitis are recommended to undergo regular surveillance colonoscopies to monitor for dysplasia, which is a precursor to cancer.
  • Crohn's disease (b) is also a type of inflammatory bowel disease that can increase the risk of colorectal cancer, especially in patients with long-standing and extensive disease.

7-17 (113C21) Which signs are more common in acute appendicitis? Choose two.

a) Blumberg's sign

b) Courvoisier's sign

c) Grey-Turner's sign

d) Murphy's sign

e) Rovsing's sign

 

The correct answers are:

a) Blumberg's sign

e) Rovsing's sign

Explanation:

  • Blumberg's sign, also known as rebound tenderness, is a clinical sign indicative of peritonitis, including that which is associated with acute appendicitis. It is observed when a sudden release of pressure on the abdomen causes pain, suggesting peritoneal irritation.

  • Rovsing's sign is specifically associated with acute appendicitis. It is elicited by pressing on the left lower quadrant of the abdomen, which may cause pain in the right lower quadrant, indicating the presence of appendicitis due to the referred pain mechanism.

  • Courvoisier's sign (b) is the presence of a palpable, non-tender gallbladder in patients with jaundice, more commonly associated with pancreatic cancer or biliary tract obstruction, not appendicitis.

  • Grey-Turner's sign (c) refers to bruising of the flanks, indicating retroperitoneal hemorrhage, and is not associated with acute appendicitis.

  • Murphy's sign (d) is used to diagnose acute cholecystitis, not appendicitis. It is positive when the patient abruptly stops inhaling due to pain when the examiner's fingers are placed under the right rib cage during inspiration.

Clinical

8-1 (100F28) A 21-year-old man presented to the hospital with diarrhea and bloody stools. For two months, he experienced mild fever, loose stools, and fatigue but did not seek medical attention due to his busy study schedule. In the past two days, he developed a fever of 37°C and had loose stools mixed with blood three to four times a day.

Which is the most appropriate initial treatment?

a) Administration of acid secretion inhibitors

b) Antimicrobial administration

c) Sulfasalazine administration

d) Corticosteroid administration

e) Central venous nutrition

The correct answer is:

c) Sulfasalazine administration

Explanation:

 

  • Sulfasalazine (Salazosulfapyridine) is commonly used in the initial treatment of inflammatory bowel diseases like ulcerative colitis, which may present with symptoms of diarrhea, bloody stools, and fever. It has both anti-inflammatory and antibacterial properties, making it suitable for reducing inflammation and controlling symptoms.

  • Administration of acid secretion inhibitors (a) is more typically used for conditions involving excess stomach acid, such as GERD or peptic ulcers, and would not be the first line of treatment for symptoms indicating an inflammatory bowel disease.

  • Antimicrobial administration (b) could be considered if an infectious cause is suspected; however, without a clear diagnosis of an infectious agent, it may not be the initial choice.

  • Corticosteroid administration (d) is used in cases of moderate to severe inflammation when other treatments have not been effective. Steroids are potent anti-inflammatory medications but due to their side effects, they are not usually the first line of treatment.

  • Central venous nutrition (e) may be necessary in severe cases where malnutrition occurs, or the gastrointestinal tract needs to rest. However, it's not the first-line treatment for initial symptoms of inflammatory bowel disease.

8-2 (100H37) A 65-year-old man experienced sudden lower abdominal pain in the middle of the night, followed by bright red melena, and was subsequently taken to the hospital by ambulance. His pulse rate was 92/min, and his blood pressure was 110/70 mmHg. He appeared pale, and his stool was bright red. Laboratory findings showed red blood cells at 3.6 million, hemoglobin at 10.0 g/dL, white blood cells at 9,000, platelets at 150,000, total protein at 7.2 g/dL, albumin at 3.5 g/dL, total bilirubin at 1.0 mg/dL, AST at 15 units, and ALT at 30 units. A colonoscopy image of the sigmoid colon is provided below.

Which statement is correct for this condition?

a) The enematogram shows an apple-core sign.

b) It is prone to occur in the left colon.

c) It is indicated for treatment with corticosteroid drugs.

d) It is prone to recurrence.

e) The prognosis is extremely poor.

The correct answer is:

b) It is prone to occur in the left colon.

Explanation:

  • Prone to occur in the left colon is correct because sigmoid colon diseases, including ischemic colitis, often present with symptoms such as lower abdominal pain and bright red blood in the stool, as described in the scenario. The left colon is a common site for the development of ischemic colitis, which can lead to such symptoms.

  • An enematogram showing an apple-core sign (a) is characteristic of colorectal cancer, specifically seen in radiographic imaging, not of ischemic colitis.

  • Corticosteroid drugs (c) are not typically indicated for ischemic colitis; they are more commonly used for inflammatory conditions like ulcerative colitis.

  • Ischemic colitis is not prone to recurrence (d).

  • The prognosis is good (e).

8-3 (100I28) A 13-year-old boy presented to the hospital complaining of bloody stools and fever. Over the past three months, he experienced approximately five episodes of diarrhea daily. For the past week, he noticed blood in his stool and has had a fever since the previous day. He has lost 3 kg over the last month. His current measurements are as follows: height 152 cm, weight 36 kg, body temperature 38.8°C, and pulse rate 120/min. Stool bacteriological tests only detected resident bacteria, and the fecal Clostridium antigen test was negative. His blood tests showed 3.5 million red blood cells, hemoglobin of 9.5 g/dL, leukocytes at 11,500, platelets at 450,000, total protein of 6.5 g/dL, albumin of 3.5 g/dL, urea nitrogen of 28 mg/dL, creatinine of 0.4 mg/dL, and C-reactive protein (CRP) of 8.5 mg/dL.

Which of the following findings is expected from the colonoscopy?

a) Pseudomembrane formation

b) Longitudinal ulcer

c) Paving stone appearance

d) Punched-out ulcer

e) Vascular ectasia

The correct answer is:

e) Vascular ectasia

Explanation:

  • Vascular ectasia, also known as vascular fluoroscopy in this context, refers to the visualization of blood vessels or vascular patterns, which can be indicative of ulcerative colitis.

  • The other options (a-d) are typically associated with various forms of gastrointestinal pathology, such as Crohn's disease (b, c) or conditions like pseudomembranous colitis (a) and specific types of ulcers (d).

8-4, 5, 6 (102B51, 102B52, 102B53) A 65-year-old man presented to the hospital with complaints of blood in his stool. Medical history: Persistent changes in bowel habits and thinning of the stools have been occurring for 2 months, with the appearance of bright red melena starting one week ago. Past medical history: None significant. Symptoms: Height 168 cm, weight 61 kg, body temperature 37.2°C, pulse 72 bpm, blood pressure 120/62 mmHg. The eyelid conjunctiva is pale, indicating anemia. Laboratory findings: Blood counts reveal erythrocytes at 3.1 million, Hb at 9.1 g/dL, Hct at 27%, leukocytes at 9,200, and platelets at 210,000. Biochemical tests show total protein at 6.2 g/dL, albumin at 3.3 g/dL, AST at 25 IU/L, ALT at 25 IU/L. Immunological findings: CRP at 2.8 mg/dL, CEA at 7.6 ng/mL (standard is 5 or less). Barium enema contrast images (A and B) are provided below.

Which diagnostic tests are useful? Choose two.

a) Small bowel follow-through

b) Abdominal CT with contrast

c) Celiac artery angiography

d) Colonoscopy

e) Plain abdominal X-ray

 

The correct answers are:

b) Abdominal CT with contrast

d) Colonoscopy

Explanation:

  • Abdominography CT scan of the abdomen can provide detailed images of the colon and can identify masses, bleeding, or other abnormalities that may cause blood in the stool.
  • Colonoscopy enables a direct visualization of the interior of the colon using a camera and can provide definitive diagnosis by visualizing the source of bleeding and allowing for biopsies of suspicious areas.

What are potential causes of melena?

a) Anal fistula

b) Rectal cancer

c) Crohn's disease

d) Colon diverticulitis

e) Ulcerative colitis

 

The correct answer is:

b) Rectal cancer

Explanation:

  • Rectal cancer can cause Melena. Melena is characterized by black, tarry stools, is often caused by bleeding higher up in the gastrointestinal tract, such as from a cancerous lesion, which could include rectal cancer.

Which treatment is appropriate?

a) Surgery

b) Antibiotic therapy

c) Immunosuppressive therapy

d) Endoscopic mucosal resection

e) Corticosteroid therapy

 

The correct answer is:

a) Surgery

Explanation:

  • For significant bleeding that is localized to a particular area, such as a cancerous growth, surgery may be necessary to remove the affected portion of the colon.

8-7 (103D37) A 17-year-old girl presented to the hospital with symptoms of diarrhea and anal pain. She has experienced a low-grade fever and loose stools for the past six months and has had frequent diarrhea, occurring six to seven times daily, for the past two weeks. Her current body temperature is 37.6°C. There is evidence of a fistula and swelling around the anus. Laboratory findings reveal anemia with a hemoglobin level of 9.1 g/dL and elevated white blood cells and C-reactive protein levels.

On lower gastrointestinal endoscopy, which two findings are most likely to be observed?

a) Cobblestone appearance

b) Pseudomembranous formation

c) C-shaped or circumferential ulcer

d) Skip lesions

e) Prominent vascular pattern

The correct answers are:

a) Cobblestone appearance

d) Skip lesions

Explanation:

  • "Cobblestone appearance" describes the mucosal pattern that resembles cobblestones, often seen in Crohn's disease due to the patchy areas of inflammation and ulceration.
  • Skip lesions are areas of diseased intestine interspersed with healthy segments, which are characteristic of Crohn's disease.

Given the symptoms of chronic diarrhea, anal fistula, and swelling, along with the blood findings, the most likely diagnosis is Crohn's disease, which is associated with both a cobblestone appearance and skip lesions observable during endoscopy. Pseudomembranous formation (b) is generally associated with Clostridioides difficile infection, which doesn't fit the presented clinical picture. C-shaped or circumferential ulcers (c) are not typically described with Crohn's disease, and prominent vascular patterns (e) are not a feature of inflammatory bowel diseases on endoscopy. Therefore, the answers would be a) Cobblestone appearance and d) Skip lesions.

8-8 (103I44) A 67-year-old woman presented to the hospital with melena. She had sporadically experienced lower abdominal pain for five days, but the previous night, she suddenly passed dark red melena. On examination, her abdomen was flat, soft, and non-tender. Laboratory findings revealed a red blood cell count of 4.22 million, hemoglobin level of 13.2 g/dL, hematocrit at 43%, a white blood cell count of 10,200, and platelets at 380,000. Total cholesterol was 202 mg/dL, total bilirubin was 1.2 mg/dL, AST at 22 IU/L, ALT at 30 IU/L, LDH at 286 IU/L (standard range 176-359), and CRP at 1.4 mg/dL.

What are the potential causes of melena?

a) Diverticulum of the large intestine

b) Infectious enteritis

c) Ulcerative colitis

d) Intestinal tuberculosis

e) Colorectal adenoma

The correct answer is:

a) Diverticulum of the large intestine

Explanation:

  • Melena is commonly caused by bleeding in the upper gastrointestinal tract but can also occur with bleeding in the right colon or ileum if the blood has time to be digested and convert to melena. A diverticulum in the large intestine is known to be a common cause of bleeding in the lower gastrointestinal tract, which can present as melena if the bleeding site is proximal enough for the blood to be digested during its passage through the intestines.

8-9, 10, 11 (104G58, 104G59, 104G60) An 84-year-old man presented to the hospital with pain in both legs. Medical History: For the past three years, he has experienced leg pain when walking long distances and has needed to take breaks. The distance he can walk without stopping has gradually decreased. Currently, he can walk only 300 meters to his field with two breaks. Past Medical History: He has been managing diabetes with diet and oral medication since the age of 50. Social History: He smoked 20 cigarettes a day for 30 years, starting at age 20, but has since quit. Clinical Findings: The patient is alert, with a height of 160 cm, weight of 51 kg, body temperature of 36.4°C, pulse rate of 72/min, and blood pressure of 154/82 mmHg. Anemia is noted in the eyelid conjunctiva. There are no abnormalities in the ocular conjunctiva. A grade 2/6 systolic murmur is heard at the heart's apex. Respiratory sounds are normal. The abdomen is soft and flat without palpable liver or spleen enlargement, and there is no edema on the anterior tibia or dorsum of the feet. Laboratory Results: Urinalysis: protein negative, glucose 2+. Hematology: red blood cells 3.19 million, hemoglobin 7.1 g/dL, hematocrit 24%, white blood cells 5,800, platelets 140,000. Biochemistry: glucose 134 mg/dL, HbA1c 7.2%, total protein 6.2 g/dL, albumin 3.7 g/dL, urea nitrogen 25 mg/dL, creatinine 0.9 mg/dL, uric acid 6.4 mg/dL, cholesterol 146 mg/dL, triglycerides 66 mg/dL, total bilirubin 0.3 mg/dL, direct bilirubin 0.1 mg/dL, AST 17 IU/L, ALT 9 IU/L, LDH 186 IU/L, ALP 242 IU/L, sodium 139 mEq/L, potassium 4.4 mEq/L, chloride 107 mEq/L, and CRP 1.3 mg/dL. Lower limb magnetic resonance angiography (MRA) images (A, B) and lumbar spine magnetic resonance imaging (MRI) images (C, D) are included.

Where are the changes occurring in relation to the chief complaint?

a) Kidney

b) Lumbar spine

c) Spinal cord

d) Arteries

e) Muscles

 

The correct answer is:

b) Lumbar spine

Explanation:

  • Given the patient's symptoms of leg pain exacerbated by walking and the recent significant change in walking distance, combined with the provided images (MRA and MRI), the changes occurring in relation to the chief complaint are most accurately associated with the lumbar spine. This suggests that lumbar spine pathology, possibly lumbar spinal stenosis or degenerative changes, could be contributing to his symptoms through neurogenic claudication, which is consistent with the described pain pattern.

Tests were performed to determine the cause of anemia, and a tumor was identified. Where is the lesion site?

a) Ascending colon

b) Descending colon

c) Rectum

d) Ureters

e) Bladder

The correct answer is:

a) Ascending colon

Explanation:

  • Since a tumor was identified as a cause of anemia, which often presents with melena or hematochezia, the lesion site is likely in the gastrointestinal tract. The most common sites for colorectal cancer, which often presents with anemia, are the ascending and descending colon, and the rectum. A guess would be the ascending colon if the anemia is significant due to the right-sided lesions often causing more pronounced blood loss.

Which is the appropriate response to this tumor?

a) Follow-up

b) Surgery

c) Radiation therapy

d) Endoscopic treatment

e) Anti-cancer chemotherapy

 

The correct answer is:

b) Surgery

Explanation:

  • The appropriate response to a tumor, depending on its stage and location, can include surgery for resection if it is deemed operable. This would often be the first line of treatment for a resectable colorectal tumor, followed by adjuvant therapy if necessary based on the tumor staging and pathology findings.

8-12 (105A22) A 55-year-old man presented to the hospital with complaints of abdominal discomfort. For the past two months, he has intermittently experienced discomfort in the lower right abdomen. No abnormalities were found upon visual examination and auscultation of the abdomen.

Which of the following is the most likely diagnosis?

a) Intussusception

b) Ascending colon cancer

c) Abdominal aortic aneurysm

d) Abdominal wall desmoid

e) Abdominal wall scar hernia

 

The correct answer is:

b) Ascending colon cancer

Explanation:

  • Ascending colon cancer can present with vague, nonspecific symptoms like intermittent abdominal discomfort. In the absence of palpable masses and given the location of discomfort (lower right abdomen), this diagnosis is plausible and aligns with the chronic nature of the symptoms described. Colon cancer in its early stages might not show any abnormalities upon basic physical examination, making it a conceivable diagnosis for the described situation.
  • Intussusception (a) is more commonly seen in children and presents with acute, severe abdominal pain, often with a palpable sausage-shaped mass, which does not align well with the described chronic, intermittent discomfort in an adult without palpable abnormalities.

  • Abdominal aortic aneurysm (c) typically presents in the mid to lower abdomen and is often asymptomatic until rupture, at which point it would cause severe pain, not intermittent discomfort. Additionally, it is more likely to be detected upon physical examination as a pulsatile mass in thin patients or through imaging studies.

  • Abdominal wall desmoid (d) and abdominal wall scar hernia (e) could potentially cause discomfort but would likely be palpable or visually apparent upon examination, especially if they were significant enough to cause symptoms.

8-13 (105B40) A 58-year-old woman presented to the hospital complaining of abdominal pain. She underwent gastrectomy two years prior and had been in good health until recently. The abdominal pain started suddenly last night and has been increasing in intensity at intervals. On examination, her consciousness was clear, her height is 155 cm, her weight is 48 kg, her body temperature is 36.8°C, her pulse is 96 beats per minute, and her blood pressure is 112/84 mmHg. The abdomen appears slightly swollen with tenderness observed throughout, but neither Blumberg's sign nor muscular defense was noted. The liver and spleen were not palpable, and an enhanced intestinal murmur was present. Urinalysis was negative for protein and glucose. Blood tests showed a red blood cell count of 3.46 million, hemoglobin of 9.7 g/dL, hematocrit of 28%, a white blood cell count of 9,100, and platelets at 160,000. Biochemical findings included a blood glucose level of 106 mg/dL, total protein of 7.1 g/dL, albumin of 4.0 g/dL, urea nitrogen of 19 mg/dL, creatinine of 1.1 mg/dL, total cholesterol of 211 mg/dL, total bilirubin of 1.0 mg/dL, AST of 35 IU/L, ALT of 38 IU/L, LDH of 346 IU/L (standard 176-353), ALP of 224 IU/L (standard 115-359), sodium of 134 mEq/L, potassium of 4.1 mEq/L, and chloride of 96 mEq/L. CRP was 1.2 mg/dL. An abdominal radiograph was also performed.

Which of the following is the appropriate response?

a) Blood transfusion

b) Laparocentesis

c) Endoscopic hemostasis

d) Catheter embolization

e) Gastrointestinal pressure decompression treatment

The correct answer is:

e) Gastrointestinal pressure decompression treatment

Explanation:

  • Gastrointestinal pressure decompression treatment seems most appropriate given the symptoms of sudden abdominal pain, swelling, and enhanced intestinal murmurs, which could suggest a partial bowel obstruction or other post-surgical complications affecting the GI tract. This approach aims to relieve the pressure within the gastrointestinal system, which can alleviate symptoms and prevent further complications.
  • Blood transfusion (a) would be indicated for significant blood loss or anemia. Although the patient's hemoglobin is on the lower side, there's no direct indication of current bleeding necessitating a transfusion based solely on the information given.

  • Laparocentesis (b) involves the removal of fluid from the peritoneal cavity, which might not be directly relevant unless there's evidence of ascites or infection needing diagnostic analysis or therapeutic intervention. The symptoms described don't directly indicate ascites.

  • Endoscopic hemostasis (c) is utilized to stop bleeding within the gastrointestinal tract. This treatment might not be relevant unless there's specific evidence of ongoing GI bleeding, which is not clearly indicated in the scenario.

  • Catheter embolization (d) is a treatment typically reserved for controlling bleeding in certain conditions, such as gastrointestinal bleeding from a specific source or vascular anomalies. There's no mention of a condition requiring such intervention.

8-14 (105E51) A 47-year-old woman visited the hospital due to diarrhea and weight loss. She underwent three partial small bowel resections for Crohn's disease 6 years ago, leaving about 90 cm of the jejunum and 20 cm of the terminal ileum. Approximately 2 years ago, she switched to an oral diet at her request and has been taking mesalazine. She sought medical attention after experiencing diarrhea more than 10 times a day for the past two weeks and losing about 3 kg in weight during this period. Her blood pressure was 102/52 mmHg. The abdomen was not tender, but intestinal murmurs were enhanced. Blood tests showed erythrocytes at 3.23 million, Hb at 11.4 g/dL, Ht at 34%, leukocytes at 5,200, and platelets at 170,000. Biochemical tests revealed albumin at 3.2 g/dL, urea nitrogen at 20 mg/dL, creatinine at 0.8 mg/dL, AST at 26 IU/L, ALT at 38 IU/L, ALP at 863 IU/L (standard 115-359), Na at 138 mEq/L, K at 3.2 mEq/L, Cl at 108 mEq/L, and Ca at 8.0 mg/dL. CRP was 0.6 mg/dL.

Which is the appropriate first step in nutritional management?

a) Oral intake of normal food

b) Oral intake of liquid foods containing dairy products

c) Enteral nutrition with elemental nutrition

d) Peripheral venous nutrition

e) Central venous nutrition

 

The correct answer is:

c) Enteral nutrition with elemental nutrition

Explanation:

 

Given the patient's extensive history of Crohn's disease and multiple partial small bowel resections, maintaining nutritional status is critical. Enteral nutrition with elemental nutrition is specially designed to be easily absorbed in the digestive tract, which is beneficial for a patient with limited intestinal capacity and active symptoms of Crohn's disease. Elemental diets contain nutrients in their simplest form, requiring minimal digestive work, making them a suitable option for patients who have undergone significant bowel resections. This approach supports intestinal healing, reduces the burden on the digestive system, and can help manage Crohn's disease symptoms more effectively than standard oral diets or other forms of nutrition that might stress the remaining bowel segments. Elemental enteral nutrition can be a pivotal part of managing Crohn's disease in patients with significant bowel alterations, providing essential nutrients while potentially reducing inflammation and promoting remission.

8-15, 16, 17 (107B58, 107B59, 107B60) A 57-year-old man visited the hospital for further evaluation after an abnormal fecal occult blood test result. At 50 years old, he had a colon polyp removed endoscopically. Since he experienced no symptoms afterward, he did not seek follow-up. However, after a colleague was diagnosed with colorectal cancer, he grew concerned about his own health and decided to consult a clinic nearby.

Past medical history includes an appendectomy at 28 years old.

Lifestyle history consists of smoking a pack of cigarettes per day for 25 years and consuming 350 mL of beer daily for 35 years, although he quit drinking two years ago.

Family history is significant for a father who died of colon cancer at 89 years old.

Currently, the patient is 165 cm tall, weighs 67 kg, and has a body temperature of 36.6°C. His pulse is 72/min, blood pressure is 130/84 mmHg, and his respiratory rate is 14/min. He has slight tenderness and post-surgical scarring in the lower right abdomen but no other noteworthy symptoms.

Laboratory tests reveal a red blood cell count of 4.2 million, hemoglobin level of 13.4 g/dL, hematocrit of 42%, white blood cell count of 8,200, and platelet count of 280,000. Blood biochemistry shows a total protein level of 7.2 g/dL, albumin level of 3.8 g/dL, total cholesterol of 230 mg/dL, AST of 36 IU/L, ALT of 36 IU/L, and a CRP of 0.03 mg/dL.

After considering the patient’s clinical history and initial findings, a lower gastrointestinal endoscopic examination was performed. The endoscopic image of the descending colon is provided below.

What is the likely diagnosis?

a) Ulcerative colitis

b) Ischemic colitis

c) Advanced colorectal cancer

d) Diverticulum of the large intestine

e) Colon polyps

 

The correct answer is:

e) Colon polyps

Explanation:

 

The endoscopic image shows a growth in the descending colon, which appears to be a polyp. Given the patient's history of previously resected colon polyps and the appearance of the lesion, it is plausible to consider this as a recurrence or a new polyp rather than a case of advanced colorectal cancer, which would typically present with more invasive features.

 

What is the most suitable treatment?

a) Laparotomy

b) Clipping

c) Endoscopic resection

d) Laparoscopic colectomy

e) Administration of mesalazine (5-ASA preparation)

 

The correct answer is:

c) Endoscopic resection

Explanation:

 

Endoscopic resection is a minimally invasive procedure that is commonly used for the removal of polyps detected during a colonoscopy. This would be appropriate here, given that the lesion is within the reach of endoscopic instruments and the patient has a history of successful endoscopic resection.

 

The patient inquired, "Since this condition is on the rise in Japan, what is believed to be the cause?" What is the accurate response?

a) "It's attributed to chronic inflammation."

b) "It's due to immunodeficiency."

c) "It's linked to viral infections."

d) "It's related to mental stress."

e) "It's associated with Western dietary habits."

 

The correct answer is:

e) "It is said to be a Westernized eating habit."

Explanation:

 

The increase in colorectal cancer in Japan has been associated with Westernized dietary habits, including increased consumption of red meat, processed foods, and decreased fiber intake. This change in eating patterns is a significant risk factor for the development of colorectal polyps and cancer. The other options listed do not have a direct established link to the increase in colorectal cancer incidence in Japan.

8-18 (108A28) A 70-year-old woman presented to the hospital with pain in the left upper abdomen. Last night, she enjoyed an energetic evening playing and singing with her grandchildren, something she had not done in a while. Three hours later, she began to experience pain in her left upper abdomen. She entered the examination room slouched over. Her appetite is good, with no nausea or vomiting, and she has regular bowel movements. She suffered a stroke three years ago and has been on aspirin therapy since then. Her body temperature is 36.5°C, pulse rate is 88/min, and blood pressure is 140/90 mmHg. She has localized tenderness in the upper left abdomen but no rebound pain.

What is the most likely diagnosis?

a) Acute pancreatitis

b) Abdominal wall hematoma

c) Iliopsoas abscess

d) Ischemic colitis

e) Perforated gastric ulcer

 

The most conceivable diagnosis given the scenario is:

b) Abdominal wall hematoma.

Explanation:

  • The patient experienced abdominal pain after a night of increased physical activity (playing and singing with grandchildren), which could potentially lead to strain and bruising, especially since she is on aspirin, which is an anticoagulant. This medication can increase the risk of bleeding, making her more susceptible to hematoma formation.
  • The pain started a few hours after the activity, which aligns with the development of a hematoma as it can take time for the bleeding to become substantial enough to cause noticeable pain.
  • Her presentation in a slouched position suggests that she is trying to minimize the stretching of the abdominal wall, which would be consistent with pain from a superficial source like a hematoma.
  • The lack of nausea, vomiting, and normal bowel movements makes gastrointestinal causes like acute pancreatitis (a), ischemic colitis (d), and a perforated gastric ulcer (e) less likely.
  • The localization of tenderness in the upper left abdomen without rebound pain is more indicative of a localized issue with the abdominal wall rather than a deep intra-abdominal or retroperitoneal process, such as an iliopsoas abscess (c).

Therefore, an abdominal wall hematoma is a plausible diagnosis, which would need to be confirmed with an appropriate imaging study, such as an ultrasound or CT scan of the abdominal wall.

8-19 (108B43) A 67-year-old woman presented to the hospital complaining of abdominal pain. Earlier today, she underwent an endoscopic polypectomy of the large intestine at a nearby clinic. After returning home, she experienced sudden pain in the lower left quadrant of her abdomen around midnight, and because the pain persisted for approximately two hours, she sought care at the emergency department. Physical examination revealed a flat abdomen with localized tenderness in the lower left quadrant.

In addition to conducting a blood test, which of the following should be performed first?

a) Abdominal MRI

b) Enema radiography

c) Exploratory laparotomy

d) Thoracoabdominal radiography

e) Lower gastrointestinal endoscopy

 

The correct answer, given the clinical scenario, is:

d) Thoracoabdominal radiography

Explanation:

  • The patient has recently undergone an endoscopic polypectomy, which carries a risk of perforation. The sudden onset of localized pain after the procedure is concerning for this complication.
  • Thoracoabdominal radiography, commonly known as an upright or supine abdominal x-ray, can be quickly performed in the emergency room and is a standard initial imaging study for suspected gastrointestinal perforation. It can reveal free air under the diaphragm, which is indicative of perforation.
  • MRI (a) and enemaography (b) are not first-line investigations in an acute setting where perforation is suspected due to their less immediate availability and the time they take to perform.
  • Test laparotomy (c) is an invasive surgical procedure and would not be the first step without imaging evidence to support the need for surgery.
  • Repeating a lower gastrointestinal endoscopy (e) immediately post-polypectomy with the presentation of acute pain would be inappropriate without first ruling out a perforation through imaging.

Therefore, the first step after a blood test should be an abdominal x-ray to check for signs of perforation, such as free intraperitoneal air. If perforation is confirmed, the patient may require urgent surgical intervention.

8-20 (108D24) A 67-year-old man presented to the hospital complaining of persistent and severe abdominal pain. He has been managed for ischemic heart disease and atrial fibrillation for the past three years. The abdominal pain started suddenly 10 hours ago and has been intensifying. His body temperature is 36.7°C, with a pulse rate of 88 beats per minute and irregular rhythm. Blood pressure is 124/78 mmHg, with a respiratory rate of 16 breaths per minute and an oxygen saturation of 97% on room air. The abdomen is distended, bowel sounds are absent, and there is tenderness with positive Blumberg's sign throughout the abdomen. Laboratory findings show a red blood cell count of 5.12 million, hemoglobin 16.2 g/dL, hematocrit 48%, white blood cells 12,800 with a differential count (28% band neutrophils, 46% segmented neutrophils, 2% eosinophils, 1% basophils, 6% monocytes, 17% lymphocytes), and platelets 180,000. Blood chemistry results include total protein 7.6 g/dL, albumin 4.6 g/dL, total bilirubin 0.6 mg/dL, AST 112 IU/L, ALT 35 IU/L, LDH 482 IU/L (reference range 176-353), amylase 124 IU/L (reference range 37-160), and CK 186 IU/L (reference range 30-140).

Which of the following is the most likely diagnosis?

a) Intussusception

b) Gastrointestinal perforation

c) Intestinal volvulus

d) Strangulating bowel obstruction

e) Superior mesenteric artery occlusion

Given the provided clinical scenario, the most likely diagnosis for the 67-year-old man with a history of ischemic heart disease and atrial fibrillation presenting with acute, severe, and persistent abdominal pain with a distended abdomen, absence of bowel sounds, and peritoneal signs (tenderness and Blumberg's sign positive throughout the abdomen) is:

e) Superior mesenteric artery occlusion.

Here's the reasoning:

  • The sudden onset of severe abdominal pain in combination with his cardiac history suggests an embolic event. Atrial fibrillation is a known risk factor for the formation of blood clots, which can dislodge and travel to the mesenteric arteries, causing an occlusion.
  • The absence of bowel sounds and the presence of severe pain out of proportion to the physical findings can be characteristic of mesenteric ischemia.
  • The presence of metabolic acidosis (although not directly provided, can be inferred from the clinical context and blood work abnormalities), leukocytosis with a left shift (presence of band neutrophils), and elevated lactate dehydrogenase (LDH) are also supportive of an ischemic process.
  • His vital signs do not indicate sepsis or hypovolemic shock which might be present in gastrointestinal perforation or severe infection, making options b and d less likely.

Therefore, the combination of his atrial fibrillation, acute severe abdominal pain, physical examination findings, and laboratory results point towards an acute mesenteric ischemia due to superior mesenteric artery occlusion as the most likely cause of his symptoms. Immediate vascular imaging (CT angiography) and surgical consultation are typically warranted for such a presentation.

8-21 (108D27) A 63-year-old woman was brought to the hospital due to abdominal pain. She had been experiencing persistent pain in her lower abdomen that had gradually worsened over the past two days. Today, the pain became severe, rendering her immobile, prompting an ambulance to be called. She has had intermittent lower abdominal pain for about five years, for which she had been taking oral medication from a nearby clinic, which had provided some relief. Physical examination revealed positive Blumberg's sign and muscular guarding in the abdomen. An abdominal CT scan showed free air, ascites, and thickening of the sigmoid colon walls, leading to a diagnosis of peritonitis due to colonic perforation, and emergency surgery was conducted. What caused the perforation?

a) Colorectal cancer

b) Intestinal tuberculosis

c) Diverticulitis of the colon

d) Ulcerative colitis

e) Colorectal polyposis

The correct answer, given the clinical scenario, is:

c) Diverticulitis of the colon

Explanation:

  • Diverticulitis is a common condition in older adults and often affects the sigmoid colon. It can lead to complications such as perforation, which would explain the presence of free air (indicative of a perforation in the gastrointestinal tract) and signs of peritonitis on physical examination.
  • While colorectal cancer (a) can also lead to perforation, it would be less likely to present with sudden severe pain unless there was an acute complication such as perforation. However, the history of intermittent pain managed with oral medications is more suggestive of a chronic condition like diverticulitis rather than cancer.
  • Intestinal tuberculosis (b) could cause perforation but is less common in developed countries and would typically present with systemic symptoms like weight loss and night sweats.
  • Ulcerative colitis (d) typically involves the colon in a continuous fashion starting from the rectum, and while it can lead to perforation, the presentation would usually include bloody diarrhea, which is not mentioned here.
  • Colorectal polyposis (e) is not typically associated with acute perforation.

Therefore, diverticulitis, particularly of the sigmoid colon, is the most likely cause of perforation in this case, considering the patient's age, the localization of symptoms, and the imaging findings.

8-22 (108D48) A 70-year-old woman was hospitalized with cellulitis of the right thigh 3 weeks ago. She responded well to the administration of cefazolin, but 2 days ago, she developed a fever of 38°C and experienced diarrhea 10 times a day. Blood findings: eosinophils 2%, monocytes 10%, lymphocytes 16%, platelets 190,000. Biochemical findings indicate albumin 2.8 g/dL, urea nitrogen 50 mg/dL (up from 0.8 mg/dL 5 days ago), creatinine 3.8 mg/dL, Na 138 mEq/L, K 4.7 mEq/L, Cl 109 mEq/L. Which is the most likely diagnosis?

a) Crohn's disease

b) Pseudomembranous enteritis

c) Ischemic colitis

d) Ulcerative colitis

e) Enterohemorrhagic Escherichia coli infection

The most conceivable diagnosis in this scenario is:

b) Pseudomembranous enteritis

Explanation:

  • Pseudomembranous enteritis, also known as Clostridioides difficile infection (CDI), often occurs after the administration of antibiotics, such as cefazolin in this case. The antibiotics can disrupt the normal gut flora, allowing C. difficile to proliferate and produce toxins that lead to colitis. The patient's recent antibiotic use for cellulitis, followed by the onset of fever and diarrhea, strongly suggests CDI.
  • Crohn's disease (a) and ulcerative colitis (d) are chronic inflammatory bowel diseases that typically present over a longer period and not typically triggered by recent antibiotic use.
  • Ischemic colitis (c) generally occurs in patients with risk factors for vascular disease and might not be directly related to antibiotic use.
  • Enterohemorrhagic Escherichia coli infection (e) could cause diarrhea but is less likely in this context without a history of exposure to contaminated food or water and typically does not follow antibiotic use.

Given the clinical context of recent antibiotic use followed by diarrhea and fever, pseudomembranous enteritis is the most likely diagnosis.

8-23 (109A35) A 45-year-old woman visited the hospital due to abdominal pain that started after lunch yesterday, accompanied by epigastric discomfort and nausea. By this morning, the pain had spread to the lower abdomen and gradually intensified. Walking exacerbated the pain, resonating with the abdominal wall, prompting her to seek medical attention. Her vital signs were: body temperature 37.8°C, pulse 92/min, blood pressure 112/70 mmHg, and respiratory rate 18/min. The abdomen was flat, with tenderness and recoil pain observed in the lower right abdomen. Intestinal murmurs were reduced, and the liver and spleen were not palpable. Urinalysis showed no protein or sugar, and no occult blood. Blood tests revealed: erythrocytes 4.71 million, hemoglobin (Hb) 14.5 g/dL, hematocrit (Ht) 42%, leukocytes 14,800, and platelets 320,000. Biochemical findings included: total bilirubin 1.3 mg/dL, aspartate aminotransferase (AST) 15 IU/L, alanine aminotransferase (ALT) 15 IU/L, alkaline phosphatase (ALP) 154 IU/L, gamma-glutamyl transferase (γ-GTP) 10 IU/L, amylase 35 IU/L, urea nitrogen 22 mg/dL, creatinine 0.6 mg/dL, blood glucose 112 mg/dL, and C-reactive protein (CRP) 3.4 mg/dL.

Which is the most appropriate treatment?

a) Cholecystectomy

b) Appendectomy

c) Right adnexectomy

d) Extracorporeal shock wave lithotripsy

e) Percutaneous transhepatic biliary drainage

The correct answer, given the clinical scenario, is:

b) Appendectomy

Explanation:

  • The presentation of abdominal pain that started in the epigastric area and then localized to the lower right abdomen, accompanied by nausea, tenderness, and rebound pain in the same area, suggests appendicitis. The elevated leukocyte count and CRP also support an inflammatory condition, such as appendicitis. Appendectomy is the standard treatment for appendicitis, making it the most likely correct option based on the provided symptoms and findings.

  • Options like cholecystectomy (a), right adnexectomy (c), extracorporeal shock wave lithotripsy (d), and percutaneous transhepatic biliary drainage (e) are treatments for other conditions (gallstones, ovarian issues, kidney stones, and biliary obstructions, respectively) that do not align as closely with the symptoms described.

8-24 (109D56) A 72-year-old man presented at the hospital with left lower abdominal pain and fever. He typically experiences constipation. He began experiencing left lower abdominal pain after a minor bowel movement two days prior. His blood work revealed: red blood cells at 3.82 million, hemoglobin at 12.8 g/dL, hematocrit at 35%, leukocytes at 18,300 (with 45% rod neutrophils, 32% segmented neutrophils, 2% eosinophils, 1% basophils, 6% monocytes, and 14% lymphocytes), and platelets at 210,000. His biochemical findings included total bilirubin at 0.8 mg/dL, AST at 30 IU/L, ALT at 42 IU/L, LDH at 238 IU/L (normal range 176–353), ALP at 350 IU/L (normal range 115–359), GGT at 60 IU/L (normal range 8–50), amylase at 62 IU/L (normal range 37–160), creatine kinase at 50 IU/L (normal range 30–140), urea nitrogen at 10 mg/dL, creatinine at 0.8 mg/dL, uric acid at 6.0 mg/dL, blood glucose at 110 mg/dL, total cholesterol at 210 mg/dL, triglycerides at 130 mg/dL, sodium at 140 mEq/L, potassium at 4.2 mEq/L, chloride at 97 mEq/L, and CRP at 6.5 mg/dL.

Which two treatments are most appropriate?

a) High-pressure enema

b) Antihypertensive drug administration

c) Antimicrobial administration

d) Right hemicolectomy

e) Puncture drainage

The correct answers are:

c) Antimicrobial administration

e) Puncture drainage

Explanation:

Given the clinical context, the most appropriate treatments would likely involve:

  • Antimicrobial administration is selected to address or prevent an infection, especially if there's suspicion of diverticulitis or another infectious cause of abdominal pain.
  • Puncture drainage is conducted if there's evidence of an abscess (not directly mentioned but could be inferred from fever, elevated leukocytes, and CRP). This would be a direct intervention to resolve the abscess.
  • Right hemicolectomy (d) seems less directly relevant based on the described symptoms of left lower abdominal pain (right hemicolectomy would address issues in the right colon).
  • High-pressure enema (a) and Antihypertensive drug administration (b) are less likely to be directly related to treating the cause of his symptoms based on the information provided. High-pressure enema could be contraindicated if there's a risk of perforation due to an underlying condition like diverticulitis, and antihypertensive drug administration does not address the acute issue unless hypertension is a concurrent concern needing management but is not directly related to the abdominal pain and fever.

8-25 (109I54) A 55-year-old man presented to the hospital with complaints of general malaise, weight loss, and abdominal pain. He has been managed as an outpatient for irritable bowel syndrome (IBS) for the past five years. Over the last three months, he has experienced increasing general malaise and has lost 5 kg. Despite taking oral medication for one month, his lower right abdominal pain has intensified. Recently, he has felt too depressed to work and has become isolated from his colleagues. He has no history of smoking or alcohol consumption. His physical examination revealed a height of 155 cm, weight of 49 kg, pulse rate of 84 bpm, blood pressure of 100/78 mmHg, anemic-like palpebral conjunctiva, a flat and soft abdomen with tenderness, and reduced bowel movements to three times a week without evident bloody stools. Laboratory findings included erythrocytes at 2.74 million, Hb 7.6 g/dL, Ht 22%, leukocytes 5,400, platelets 280,000, total protein 6.3 g/dL, albumin 3.6 g/dL, total bilirubin 1.0 mg/dL, AST 21 IU/L, ALT 11 IU/L, LD 179 IU/L (standard 176-353), ALP 227 IU/L (standard 115-359), γ-GTP 40 IU/L (standard 8-50), urea nitrogen 17 mg/dL, creatinine 0.9 mg/dL, and CRP 0.1 mg/dL.

Which is the appropriate response?

a) Consultation with a psychiatrist

b) Medication change for the treatment of irritable bowel syndrome

c) Search for organic diseases

d) Central venous nutrition

e) Follow-up

 

Given the symptoms and history provided, the most appropriate response seems to be:

c) Search for organic diseases

Explanation:

 

This man has been diagnosed with irritable bowel syndrome but is experiencing significant weight loss, general malaise, and worsening abdominal pain, which are concerning for potential underlying conditions not typically associated with IBS alone. The significant weight loss, in particular, warrants a thorough investigation to rule out any organic diseases such as inflammatory bowel diseases, malignancies, or other conditions that could present with these symptoms. The worsening of symptoms despite treatment for IBS, combined with new systemic symptoms like weight loss and general malaise, suggests that a reevaluation of his diagnosis and further diagnostic testing are necessary to ensure no significant underlying conditions are missed.

8-26 (110A59) A 50-year-old man visited the hospital with the primary complaint of diarrhea and mucous bloody stools persisting for 2 months, intensifying to 6 or 7 mucous stools daily over the past week. He has no history of international travel. His physical examination revealed a height of 164 cm, weight of 54 kg, body temperature of 37.8°C, pulse rate of 88/min, and blood pressure of 120/60 mmHg. The eyelid conjunctiva appeared mildly anemic. Colonoscopy showed multiple erosions and ulcers within the colon.

Which findings are expected in this case? Choose three.

a) Varicose veins

b) Crypt abscess

c) Reduction of goblet cells

d) Hyperplastic polyps

e) Diffuse inflammatory cell infiltration

 Given the described symptoms and findings, including diarrhea, mucous bloody stools, and endoscopic observation of multiple erosions and ulcers in the colon, the most appropriate findings that align with these clinical presentations would likely involve inflammatory processes, specifically:

  • b) Crypt abscess: These are indicative of severe inflammation within the colonic crypts, commonly seen in conditions like ulcerative colitis, which matches the symptoms and endoscopic findings described.

  • c) Reduction of goblet cells: Goblet cell depletion is a hallmark of chronic inflammation in the colon, as seen in inflammatory bowel diseases (IBD) like ulcerative colitis and Crohn's disease. The continuous inflammatory process leads to the destruction and reduced number of these mucus-producing cells, which can contribute to the symptoms of diarrhea and mucous bloody stools.

  • e) Diffuse inflammatory cell infiltration: This finding supports the presence of widespread inflammation in the colon, which is consistent with the symptoms and the presence of erosions and ulcers seen on endoscopy. It is characteristic of inflammatory bowel diseases.

These findings align with the clinical picture of an inflammatory bowel disease, such as ulcerative colitis, given the patient's symptoms of chronic diarrhea and bloody stools, absence of a travel history (which might suggest an infectious etiology), and the endoscopic findings of ulceration. Varicose veins (a) are more indicative of issues like portal hypertension and are unrelated to the colon's mucosal inflammation. Hyperplastic polyps (d) are typically benign and not associated with diffuse inflammatory processes or the described symptoms.

8-27 (110E54) A 78-year-old man presented at the hospital with chief complaints of nausea and abdominal pain. He described the pain as originating in the morning, gradually migrating to the lower right abdomen, and becoming more intense over time. His vital signs upon examination were as follows: height 160 cm, weight 54 kg, body temperature 37.8°C, pulse rate 92 beats per minute, blood pressure 148/84 mmHg, and a respiratory rate of 20 breaths per minute. The abdominal examination revealed a flat abdomen with tenderness and rebound pain localized to the lower right quadrant. Laboratory tests showed a red blood cell count of 3.65 million, hemoglobin (Hb) 13.2 g/dL, hematocrit (Ht) 35%, leukocyte count of 12,100 (with 10% band neutrophils, 72% segmented neutrophils, 1% eosinophils, 3% monocytes, and 14% lymphocytes), and platelet count of 190,000. Biochemical findings included urea nitrogen 18 mg/dL, creatinine 0.9 mg/dL, and C-reactive protein (CRP) 1.2 mg/dL. Abdominal radiographs showed no abnormalities.

Which test should be performed next?

a) PET/CT

b) Abdominography CT

c) Abdominal angiography

d) Capsule endoscopy

e) Lower gastrointestinal endoscopy

 

Given the clinical presentation of nausea, abdominal pain that localized to the lower right abdomen and intensified, along with tenderness and rebound pain in the same area, the described symptoms are highly suggestive of appendicitis. The elevated leukocyte count with a predominance of neutrophils and increased CRP levels further support an inflammatory condition, likely localized to the abdomen.

The most appropriate next step to confirm the diagnosis and assess the extent of the inflammation or any complications like an abscess or perforation would be:

  • b) Abdominography CT: This imaging modality is highly effective for diagnosing appendicitis, providing detailed images of the appendix and surrounding structures. It can identify signs of appendiceal inflammation, such as wall thickening, periappendiceal fat stranding, and any associated complications.

PET/CT (a) is not routinely used for diagnosing acute appendicitis due to its focus on metabolic activity and common use in oncology. Abdominal angiography (c) is primarily used for vascular conditions and not indicated in this scenario. Capsule endoscopy (d) is used for small bowel evaluation and would not be suitable for assessing appendicitis. Lower gastrointestinal endoscopy (e) is utilized for visualizing the interior lining of the colon and rectum, which would not directly visualize the appendix or effectively diagnose appendicitis.

8-28 (111A31) An 82-year-old woman was admitted to the hospital with abdominal pain and bloody stools. She noted temporary relief from abdominal pain after bowel movements, but the pain intensified this morning and was accompanied by nausea. Subsequently, she experienced continuous bloody stools, prompting her visit to the doctor. She has been treated for hypertension at a local clinic for the past 10 years. On examination, she was alert with a height of 153 cm, weight of 54 kg, body temperature of 37.2°C, pulse rate of 88 beats per minute, blood pressure of 120/84 mmHg, respiratory rate of 14 breaths per minute, and SpO2 of 98% on room air. The abdomen was flat with tenderness in the lower left abdomen, but no Blumberg's sign or muscular guarding was observed. Intestinal sounds were diminished, and no metallic sounds were heard. Laboratory tests revealed red blood cell count of 3.5 million, hemoglobin 11.0 g/dL, hematocrit 43%, leukocyte count of 9,200, and platelet count of 380,000. Biochemical findings included urea nitrogen of 19 mg/dL, creatinine of 1.2 mg/dL, and C-reactive protein (CRP) of 5.0 mg/dL. Abdominal X-rays in standing and lying positions are shown below.

Which is the appropriate response after hospitalization?

a) Decompression with ileus tube

b) Open surgery

c) Catheter embolization

d) Colonoscopy for decompression

e) Conservative treatment

The correct answer is:

e) Conservative treatment

Explanation:

  • Conservative treatment is the appropriate response after hospitalization for the described patient scenario highlights the importance of considering the overall clinical presentation and patient's condition in decision-making. Conservative treatment might involve monitoring, fluid management, and possibly antibiotics, reflecting a cautious approach to care, especially in elderly patients where the risks of invasive procedures can be significantly higher. This approach can be particularly prudent when the symptoms might resolve with non-invasive management, or in cases where the exact cause of symptoms is not clear and the patient's condition is stable enough to allow for observation and supportive care.

8-29 (111A48) A 61-year-old woman visited the hospital due to abdominal pain. She started experiencing a dull ache in her upper abdomen yesterday, which has gradually worsened. The pain is persistent, spreading from the epigastric region to the umbilical region, slightly more intense on the right side, but not well-localized. She reports nausea but no vomiting. Her vital signs are as follows: body temperature of 37.4°C, pulse rate of 72/min, blood pressure of 120/72 mmHg, and a respiratory rate of 16/min. Examination reveals tenderness and mild rebound pain in the affected area but no muscular defense. She has loose stools, not watery diarrhea, with no blood present. Intestinal sounds are slightly diminished, with no metallic sounds detected. Laboratory results show a red blood cell count of 4.32 million, hemoglobin of 13.1 g/dL, hematocrit of 39%, white blood cell count of 15,500 (with 32% band neutrophils, 58% segmented neutrophils, 1% eosinophils, 1% basophils, and 8% lymphocytes), and platelets at 290,000. Biochemical findings include a urea nitrogen level of 10 mg/dL, creatinine of 0.7 mg/dL, and C-reactive protein (CRP) of 5.7 mg/dL. Abdominal ultrasonography reveals no abnormalities in the liver and gallbladder.

Which treatment is most appropriate?

a) Antimicrobial administration

b) Colectomy

c) Appendectomy

d) Decompression with ileus tube

e) Extracorporeal shock wave lithotripsy (ESWL)

The correct answer is:

a) Antimicrobial administration

Explanation:

  • The patient presents with dull pain in the upper abdomen that is persistent and widespread, with slight localization to the right side, accompanied by nausea but no vomiting. The absence of severe symptoms such as high fever, vomiting, or signs of obstruction and the normal findings in liver and gallbladder on ultrasonography suggest that the condition might not require immediate surgical intervention or procedures like ESWL, which is typically used for kidney stones.

  • Given the elevated white blood cell count (leukocytosis with a shift to the left) and elevated CRP levels, which indicate an inflammatory or infectious process, Antimicrobial administration seems the most appropriate initial treatment. This choice suggests that the clinical picture might be due to an infectious or inflammatory condition within the abdomen that does not have an obvious surgical cause based on the information provided.

  • Colectomy (b), Appendectomy (c), and Decompression with ileus tube (d) are more invasive and would typically be considered in the presence of clear surgical indications such as confirmed appendicitis, bowel obstruction, or colon pathology. Extracorporeal Shock Wave Lithotripsy (ESWL) (e) is not indicated here due to the lack of evidence of renal stones or gallstones that warrant such treatment.

8-30 (111D48) A 78-year-old man was brought to the hospital by ambulance due to abdominal pain. He had been experiencing constipation for a year without seeking medical advice. Recently, his constipation worsened, and after taking a laxative last night, he experienced severe abdominal pain during a bowel movement this morning, prompting him to call an ambulance. His blood pressure was recorded at 160/92 mmHg, with a respiratory rate of 28/min. The patient appeared distressed and was lying in a lateral decubitus position. The conjunctiva of his eyelids showed signs of anemia, but there was no jaundice in the ocular conjunctiva. Heart and respiratory sounds were normal. His abdomen was distended and showed significant tenderness. Superficial lymph nodes were not palpable. Laboratory results revealed: erythrocytes count of 3,200,000, hemoglobin at 10.7 g/dL, hematocrit at 30%, white blood cells at 15,300, and platelets at 180,000. Biochemical tests showed: total protein at 6.6 g/dL, albumin at 3.4 g/dL, total bilirubin at 0.6 mg/dL, AST at 50 U/L, ALT at 62 U/L, LD at 330 U/L, ALP at 270 U/L, γ-GTP at 63 U/L, amylase at 140 U/L, CK at 110 U/L, urea nitrogen at 28 mg/dL, creatinine at 1.0 mg/dL, uric acid at 6.0 mg/dL, blood glucose at 130 mg/dL, HbA1c at 5.0%, total cholesterol at 178 mg/dL, triglycerides at 190 mg/dL, sodium at 142 mEq/L, potassium at 4.2 mEq/L, and chloride at 98 mEq/L, with a CRP of 11 mg/dL. An abdominal CT scan was performed.

Which treatment is most appropriate?

a) Rectal amputation

b) Closure of perforation

c) Drainage

d) Hypothermic anterior resection

e) Closure of perforation + colostomy (Hartmann surgery)

The correct answer is:

e) Closure of perforation + colostomy (Hartmann surgery)

Explanation:

 

Given the patient's severe abdominal pain, the presence of anemia, elevated inflammatory markers (CRP 11 mg/dL), and the need for an ambulance due to severe discomfort, combined with his history of constipation and recent laxative use, the clinical picture suggests an acute abdominal condition, possibly related to bowel obstruction or perforation. The abdominal CT likely plays a crucial role in identifying the exact cause of his symptoms, such as a perforation or ischemic event leading to severe abdominal pain.

Considering the options provided:

  • Rectal amputation (a) is typically indicated for rectal cancer, not acute abdominal emergencies without a diagnosed rectal malignancy.
  • Closure of perforation (b) is a plausible option if the CT shows evidence of bowel perforation, which is a common emergency surgical intervention.
  • Drainage (c) might be appropriate if there's an abscess or localized infection, but without seeing the CT, it's speculative.
  • Hypothermic anterior resection (d) is a specific surgical procedure for rectal cancer, less likely to be the first line of action in an emergency setting without a prior diagnosis of cancer.
  • Closure of perforation + colostomy (Hartmann surgery) is considered for significant colon injuries or conditions where the bowel needs time to heal, and diverting the fecal stream is necessary. This option is viable if the CT indicates severe damage to the bowel that cannot be directly repaired in a single procedure, requiring temporary diversion.

Given the acute presentation and the potential for a severe complication like perforation, the most appropriate responses, without specific details from the CT scan, could be Closure of perforation or Closure of perforation + colostomy (Hartmann surgery), depending on the extent of the damage revealed by the CT. The decision between these would depend on the specific findings of the CT scan, such as the location and extent of the perforation, the bowel's condition around the perforated area, and the patient's overall health status.

Option (e) Closure of perforation + colostomy (Hartmann surgery) might be more appropriate if there's significant concern about the viability of the bowel or if there's extensive contamination that necessitates bowel rest and diversion to allow for healing.

8-31 (111G57) An 82-year-old woman is hospitalized with pneumonia. Antibiotics have been administered, and the symptoms of pneumonia have been mild. However, she has been experiencing frequent watery diarrhea for the past three days. She is also receiving oral treatment for hypertension. She is conscious and clear-headed, with a body temperature of 37.6°C, a pulse rate of 76/min, and a blood pressure of 138/78 mmHg. Her abdomen is flat and soft, with slight tenderness observed in the lower abdomen. Blood tests show erythrocytes at 3.8 million, Hb at 12.0 g/dL, Ht at 36%, leukocytes at 9,800, and platelets at 260,000. There are no abnormal findings on abdominal radiographs, but the stool test is positive for Clostridium difficile toxin.

Which medications may be effective for this patient? Choose two.

a) Vancomycin

b) Clindamycin

c) Erythromycin

d) Metronidazole

e) Benzylpenicillin (Penicillin G)

 

The correct answers are:

a) Vancomycin

d) Metronidazole

Explanation:

 

In the scenario described, the patient has pneumonia and is experiencing frequent watery diarrhea, a common symptom of Clostridium difficile infection, which is confirmed by the positivity for Clostridium difficile toxin in the stool. The choice of antibiotics for Clostridium difficile infection (CDI) typically includes:

  • Vancomycin is one of the primary treatments for CDI. It works by inhibiting cell wall synthesis and is not absorbed in the intestine, which makes it very effective for treating infections in the gut.

  • Metronidazole was once the first-line treatment for mild to moderate CDI but is now generally considered an alternative to vancomycin, especially when vancomycin is not available, cannot be used due to allergies, or for non-severe infections.

The other antibiotics listed:

  • Clindamycin (b) is known to be one of the antibiotics associated with the development of CDI and would not be used to treat it.

  • While Erythromycin (c) has antibacterial properties, it is not typically used in the treatment of CDI.

    Benzylpenicillin (Penicillin G) (e) is not typically used to treat CDI and can be associated with the development of the infection.

8-32 (112A33) A 60-year-old woman presented at the hospital with complaints of bloody stools and abdominal pain. She has experienced chronic constipation, and her last bowel movement occurred 5 days ago. Two days ago, she began experiencing abdominal pain, followed by episodes of passing fresh blood. She is currently under treatment for dyslipidemia and diabetes. Her body temperature is 36.7°C, pulse rate is 92/min, and blood pressure is 126/84 mmHg. No anemia is apparent in the eyelid conjunctiva. The abdomen is flat and soft without palpable liver or spleen enlargement, but tenderness is noted in the lower abdomen. Blood tests reveal red blood cells count of 4.3 million, hemoglobin of 13.1 g/dL, hematocrit of 39%, white blood cells count of 8,700, and platelets count of 190,000. The CRP level is 1.2 mg/dL. Lower gastrointestinal tract endoscopy was performed, and the endoscopic image of the sigmoid colon is shown below.

What is the appropriate treatment?

a) Initiate fasting

b) Administer a corticosteroid enema

c) Perform endoscopic hemostasis

d) Embolize the superior mesenteric artery

e) Proceed with a colectomy

The correct answer is:

a) Initiate fasting

Explanation:

 

Ischemic colitis involves inflammation and injury of the large intestine resulting from inadequate blood supply. The primary treatment often includes supportive measures such as bowel rest (fasting), hydration to prevent dehydration, and careful monitoring. This approach allows the colon to heal and can help prevent further damage. Depending on the severity, additional interventions may be required, but the initial conservative management typically involves non-invasive measures.

8-33 (112D74) A 2-year-old boy was hospitalized for bronchopneumonia treatment. He received cephem antibiotics, but diarrhea developed on the 5th day of hospitalization. Despite the slight fever and diarrhea occurring several times a day, his mood remained good. His measurements were as follows: height 76.9 cm, weight 12.8 kg, body temperature 37.7°C, pulse 124/min, blood pressure 112/48 mmHg, and respiratory rate 30/min. Examination showed no abnormalities in the eyelid or ocular conjunctiva, no pharyngeal redness, normal heart and lung sounds, a flat and soft abdomen with no palpable liver or spleen, mild intestinal murmurs, no extremity edema, and a positive stool test for Clostridium difficile antigen.

For future medical examinations, which two precautions should be taken? a) Wear gloves.

b) Wear an apron.

c) Wear an N95 mask.

d) Implement negative pressure private isolation.

e) Space beds by at least 2 meters.

 

The correct actions to take during future medical examinations, considering the positive test for Clostridium difficile antigen and the need to prevent its spread, are:

a) Wear gloves. b) Wear an apron.

Explanation:

  • Wear gloves: This is essential for preventing the direct contact and spread of Clostridium difficile, as the bacteria can be transmitted via fecal-oral route. Gloves protect healthcare workers and prevent the transmission of infectious agents during patient care.

  • Wear an apron: An apron provides an additional layer of protection against possible contamination from Clostridium difficile, especially during procedures or care activities that might involve exposure to feces or bodily fluids. This precaution helps in preventing the spread of infection to healthcare workers and other patients.

Options c, d, and e, while important in certain contexts, are not specifically required for Clostridium difficile precautions according to standard infection control guidelines:
  • Wear an N95 mask (c): N95 masks are recommended for airborne precautions, which are not typically necessary for Clostridium difficile, as it is not spread through the air but rather through contact with contaminated surfaces or fecal matter.

  • Implement negative pressure private isolation (d): Negative pressure rooms are used for airborne infections (e.g., tuberculosis, measles, chickenpox), and Clostridium difficile does not require such measures as it is not airborne.

  • Space beds by at least 2 meters (e): While maintaining distance can help reduce the spread of some infections, the key to preventing Clostridium difficile transmission in a healthcare setting is meticulous attention to contact precautions, environmental cleaning, and hand hygiene, rather than spatial separation alone.

8-34 (113A56) An 81-year-old woman is hospitalized for stroke rehabilitation. She was treated with antibiotics for bacterial pneumonia for two weeks. A week ago, she started experiencing abdominal pain and diarrhea, which became more frequent as of yesterday. She is alert, with a height of 156 cm and weight of 41 kg. Her body temperature is 37.9°C, pulse is 80/min and irregular, blood pressure is 146/90 mmHg, and respiratory rate is 16/min. Her oxygen saturation is 96% on a nasal cannula at 3 L/min. No abnormalities are found in heart sounds, but coarse crackles are audible in both lungs. The abdomen is flat, with no palpable liver or spleen, and there is tenderness in the left lower abdomen. Left upper and lower limb paresis is noted. Blood analysis shows red blood cells at 3.58 million, hemoglobin at 10.9 g/dL, leukocytes at 13,300, and platelets at 190,000. Biochemical findings include a total protein of 5.7 g/dL, albumin of 2.9 g/dL, total bilirubin of 0.9 mg/dL, AST at 26 U/L, ALT at 19 U/L, LD at 245 U/L (standard 176 to 353), creatinine at 1.1 mg/dL, blood glucose at 98 mg/dL, HbA1c at 7.1% (standard 4.6 to 6.2), sodium at 138 mEq/L, potassium at 3.4 mEq/L, chloride at 101 mEq/L, and CRP at 3.1 mg/dL. There are no abnormalities on abdominal radiographs. She tested positive for Clostridium difficile toxin in her stool.

Which test should be conducted next?

a) Berotoxin

b) β-D-Glucan

c) Helicobacter pylori antigen in stool

d) Clostridium difficile toxin in stool

e) Mycobacterium tuberculosis-specific whole blood interferon-gamma release assay (IGRA)

 

The correct answer is:

d) Clostridium difficile toxin in stool

Explanation:

 

For an 81-year-old woman who has been hospitalized and treated with antibiotics, presenting with abdominal pain and diarrhea, a common concern would be Clostridium difficile infection (CDI), especially given the recent antibiotic treatment, which is a known risk factor for CDI.

  • The appropriate test to perform next would be Clostridium difficile toxin in stool. This test is specific for diagnosing CDI, which can cause symptoms ranging from mild diarrhea to severe inflammation of the colon. Given the patient's recent antibiotic use and symptoms, this would be a logical next step in her diagnostic workup.

  • Another test that might be considered, but is less directly indicated by the symptoms described, is Berotoxin (a), if there is a suspicion of other types of toxin-mediated diarrhea, although it's less common than CDI in this clinical scenario.

  • Tests like β-D-Glucan (b) (associated with fungal infections), Helicobacter pylori antigen (c) (associated with gastric ulcers), and Mycobacterium tuberculosis-specific whole blood interferon-gamma release assay (IGRA) γ (e) (used for tuberculosis screening) are not directly indicated based on the presented symptoms and recent medical history.

8-35 (113C30) A 72-year-old woman visited the emergency room for lower abdominal pain and bloody stools. She experienced sudden lower abdominal pain and diarrhea before going to bed today and sought examination due to the appearance of bright red bloody stools following several episodes of diarrhea. She has been managed for diabetes and hypertension at a clinic near her home for 20 years. She is conscious and alert, with a body temperature of 37.2°C, pulse rate of 96/min, blood pressure of 142/92 mmHg, respiratory rate of 20/min, and SpO2 of 96% on room air. Her abdomen is flat, with spontaneous pain and tenderness in the lower left abdomen, but no muscular defense. Blood tests revealed 3.8 million red blood cells, hemoglobin of 11.4 g/dL, hematocrit of 39%, leukocytes at 11,200 (with 4% band neutrophils, 55% segmented neutrophils, 2% eosinophils, 7% monocytes, 32% lymphocytes), and platelets at 230,000. Biochemical findings include total protein of 6.9 g/dL, albumin of 3.8 g/dL, total bilirubin of 0.9 mg/dL, AST of 24 U/L, ALT of 27 U/L, LD of 267 U/L (standard 176 to 353), amylase of 60 U/L (standard 37 to 160), urea nitrogen of 21 mg/dL, creatinine of 1.1 mg/dL, uric acid of 6.6 mg/dL, blood glucose of 138 mg/dL, HbA1c of 6.9% (standard 4.6 to 6.2), sodium of 141 mEq/L, potassium of 4.4 mEq/L, and chloride of 99 mEq/L. CRP is 2.1 mg/dL. Arterial blood gas analysis on room air showed pH of 7.41, PaCO2 of 36 Torr, PaO2 of 90 Torr, and HCO3- of 24 mEq/L.

Which disease is most likely?

a) Ischemic enteritis

b) Drug-induced enteritis

c) Perianal abscess

d) Eosinophilic gastroenteritis

e) Superior mesenteric artery occlusion

 

The correct answer is:

a) Ischemic enteritis

Explanation:

 

Given the clinical presentation and findings, the most likely diagnosis for a 72-year-old woman with sudden lower abdominal pain, diarrhea followed by bright red bloody stools, and a history of diabetes and hypertension, is Ischemic enteritis.

  • Ischemic enteritis occurs when there is insufficient blood flow to the intestines, leading to inflammation and injury. The sudden onset of symptoms, especially in an elderly patient with vascular risk factors such as hypertension and diabetes, supports this diagnosis. The presence of bloody stools can occur when ischemia leads to mucosal sloughing and ulceration. The abdominal pain, particularly localized in the lower abdomen without muscular defense, suggests a non-perforative condition initially, which is consistent with ischemic changes rather than a perforative disease like a perianal abscess or conditions that typically present with more diffuse symptoms like eosinophilic gastroenteritis.
  • The lack of specific drug history to suggest drug-induced enteritis (b) and the absence of clinical features pointing towards a perianal abscess (c) or eosinophilic gastroenteritis (d) further supports ischemic enteritis as the most probable diagnosis. Superior mesenteric artery occlusion (e) could also cause ischemia, but it typically presents more acutely with severe pain out of proportion to physical findings and might not specifically lead to bloody stools without significant bowel infarction, which would likely present with more severe clinical deterioration.

Hence, ischemic enteritis is the most plausible diagnosis given the patient's age, symptomatology, and risk factors.

8-36 (113C42) A 62-year-old woman came to the hospital for a detailed examination due to a positive fecal occult blood test. Reluctant to undergo endoscopy, she reports daily bowel movements without changes in stool caliber, no significant weight fluctuation in recent years, no notable medical history, occasional alcohol consumption, no smoking history, and no family history of colorectal cancer. Physical examination reveals a height of 155 cm, weight of 56 kg, a flat and soft abdomen, no palpable liver or spleen, and no abdominal tenderness.

Which is the most appropriate response?

a) "Let's perform an abdominal ultrasound."

b) "Let's conduct a blood test for tumor markers of colorectal cancer."

c) "Let's monitor the situation and retest the fecal occult blood in 6 months."

d) "Let's retest the fecal occult blood and consider the results."

e) "Let me explain the importance of undergoing a colonoscopy."

 

The correct answer is:

e) "Let me explain the importance of undergoing a colonoscopy."

Explanation:

  • Given the clinical context—a 62-year-old woman presenting for a detailed examination due to fecal occult blood, with no significant personal or family history suggesting a high risk of colorectal cancer, but hesitant to undergo endoscopy—the most appropriate response would be "Let me explain the importance of undergoing a colonoscopy."
  • A colonoscopy is the gold standard for diagnosing colorectal cancer and other conditions that may cause occult blood in the stool, such as polyps, diverticulosis, and inflammatory bowel disease. Given the patient's age and the finding of fecal occult blood, even in the absence of other symptoms or risk factors, explaining the importance and benefits of a colonoscopy for early detection and potential prevention of colorectal cancer is crucial. It can detect polyps before they turn cancerous or catch cancer at an early, more treatable stage.
  • While abdominal ultrasound (a) and tumor marker tests (b) can be helpful in some contexts, they are not substitutes for a colonoscopy in evaluating occult gastrointestinal bleeding. Option c, delaying retesting, and option d, retesting without further action, may delay diagnosis and treatment of potentially serious conditions. Therefore, educating the patient on the importance of a colonoscopy, addressing her hesitations, and offering support for the procedure is the most appropriate and responsible course of action.

8-37, 38, 39 (113C63, 113C64, 113C65) A 62-year-old man presented at the hospital with blood in his stool.

Current Medical History: He visited the emergency room after noticing a significant amount of dark red stool after dinner today, accompanied by lightheadedness.

Medical History: He has been taking warfarin for hypertension and diabetes for 30 years and for atrial fibrillation for the past 10 years.

Life History: He lives with his wife, smoked 20 cigarettes a day for 20 years until he was 50, and drinks 350 mL of beer daily.

Family History: His father had a stroke, and his mother had colon cancer.

Symptoms: He is alert, with a height of 169 cm, weight of 70 kg, body temperature of 36.7°C, pulse rate of 88/min with irregularity, blood pressure of 114/78 mmHg, respiratory rate of 18/min, and SpO2 of 96% on room air. His eyelid conjunctiva is anemic without ocular jaundice. Heart and lung sounds are normal. His abdomen is flat and soft without palpable liver or spleen, with slightly enhanced intestinal murmurs. Dark red stool was noted on digital rectal examination, and mild cold sweat was observed on his extremities.

Laboratory findings: Blood findings: erythrocytes 2.99 million, Hb 9.7 g/dL, Ht 32%, leukocytes 12,000, platelets 210,000. Blood biochemical findings: total protein 6.5 g/dL, albumin 3.6 g/dL, total bilirubin 0.9 mg/dL, AST 28 U/L, ALT 22 U/L, LD 277 U/L (standards 176-353), γ-GTP 41 U/ L (criteria 8 to 50), amylase 80 U/L (criteria 37 to 160), urea nitrogen 18 mg/dL, creatinine 1.1 mg/dL, uric acid 6.7 mg/dL, blood glucose 128 mg/dL, Na 140 mEq/L, K 4.5 mEq/L, Cl 100 mEq/L, CRP 1.9 mg/dL.

Abdominal simple CT (A) and abdominal contrast CT (B, C) are shown. below.

Which one should be measured first?

a) PaO2

b) PT-INR

c) D-dimer

d) Platelet adhesion

e) Myocardial Troponin T

 

The correct answer is:

b) PT-INR

Explanation:

 

Given his medical history and current presentation, particularly his use of warfarin, PT-INR (Prothrombin Time-International Normalized Ratio) should be measured first to assess his coagulation status and the effect of warfarin, as warfarin use significantly increases the risk of bleeding, and an elevated PT-INR would indicate an increased risk.

 

Which is the most conceivable diagnosis?

a) Diverticulosis of the colon

b) Ischemic enteritis

c) Ulcerative colitis

d) Non-obstructive intestinal ischemia

e) Enterohemorrhagic Escherichia coli infection

 

The correct answer is:

a) Diverticulosis of the colon

Explanation:

 

Considering his symptoms and clinical presentation, diverticulosis of the colon is the most conceivable diagnosis. Diverticulosis can cause significant bleeding in the colon, and the history of hypertension and use of warfarin further predispose him to bleeding risks. Ischemic enteritis (b) and non-obstructive intestinal ischemia (d) are less likely without evidence of vascular compromise on imaging, and ulcerative colitis (c) typically presents with a longer history of symptoms. Enterohemorrhagic Escherichia coli infection (e) is unlikely without relevant exposure history and would usually present with a different clinical picture.

 

Subsequent Progress: After hospitalization, a transfusion of 6 units of RBC-LR was administered by the next morning, but the hemoglobin level was 8.2 g/dL, and blood in the stool persisted.

Which actions should be considered at this point? Choose two.

a) Bowel resection

b) Arterial embolization

c) Thrombolytic therapy

d) Plasmapheresis

e) Hyperbaric oxygen therapy

 

The correct answers are:

a) Bowel resection

b) Arterial embolization

Explanation:

 

Given the ongoing blood loss despite transfusion, indicating active bleeding that hasn't been controlled, the two considerations should include:

  • Bowel resection may be considered if a specific bleeding source within the bowel is identified that cannot be controlled by less invasive means.
  • Arterial embolization is an effective, minimally invasive procedure to control bleeding, particularly if the source is from a diverticulum or a specific vessel, without requiring full surgery.
  • Thrombolytic therapy (c), plasmapheresis (d), and hyperbaric oxygen therapy (e) are not standard treatments for gastrointestinal bleeding and do not address the most likely causes of his symptoms based on the information provided.

8-40 (114D25) A 78-year-old man presented at the hospital experiencing abdominal pain. The pain emerged suddenly four hours prior and has progressively worsened, prompting his visit for evaluation. He has been managing atrial fibrillation with oral medication for the past two years. His vital signs include a body temperature of 37.1°C, a pulse of 120 beats per minute with an irregular rhythm, and a blood pressure of 86/56 mmHg. His respiratory rate is 24 breaths per minute. The patient's abdomen is distended with generalized tenderness upon examination. Laboratory results reveal a red blood cell count of 5.1 million, hemoglobin at 15.8 g/dL, hematocrit at 45%, a white blood cell count of 9,500, and platelets at 130,000. Biochemical findings show total protein at 6.8 g/dL, albumin at 3.4 g/dL, AST at 16 U/L, ALT at 14 U/L, lactate dehydrogenase (LDH) at 310 U/L (standard range 120-245), creatine kinase (CK) at 275 U/L (standard range 30-140), blood urea nitrogen (BUN) at 31 mg/dL, creatinine at 0.8 mg/dL, sodium (Na) at 134 mEq/L, potassium (K) at 5.2 mEq/L, and chloride (Cl) at 108 mEq/L. An abdominal contrast-enhanced CT scan has been provided below for further evaluation.

What is the most appropriate course of treatment?

a) Perform an emergency laparotomy

b) Initiate hyperbaric oxygen therapy

c) Attempt endoscopic reduction

d) Conduct embolization of the superior mesenteric artery

e) Place a transanal ileus tube

The correct answer is:
a) Perform an emergency laparotomy
Explanation:

 

Given the presentation of sudden abdominal pain, hypotension, and tachycardia, a serious intra-abdominal event such as ischemia or perforation must be considered, and an emergency laparotomy could be the appropriate intervention to diagnose and treat the cause of the pain. The other options might be considered after specific diagnoses are made with the help of imaging studies and clinical correlation.

8-41 (114D45) A 50-year-old man was admitted to the hospital with the primary complaint of pain in the lower right abdomen. While working on a ship, he experienced pain in the lower right abdomen and a fever seven days ago and visited a hospital at the next port. There, he was diagnosed with acute appendicitis and admitted for treatment. He received intravenous fluids and antibiotics through a peripheral vein while fasting. By the third day of his stay, his fever subsided and the pain in his lower abdomen decreased. He was referred to our hospital for continued treatment closer to home. Upon examination, the patient was alert and oriented, standing at 173 cm tall and weighing 70 kg. His body temperature was 36.2°C, and his abdomen was flat, soft, and non-tender with normal bowel sounds. Laboratory results were as follows: red blood cells 4,860,000, hemoglobin 14.8 g/dL, hematocrit 43%, white blood cells 6,400, platelets 210,000. Biochemical tests revealed total protein at 6.4 g/dL, albumin at 3.2 g/dL, total bilirubin at 0.7 mg/dL, AST at 14 U/L, ALT at 14 U/L, blood urea nitrogen at 19 mg/dL, creatinine at 0.8 mg/dL, and C-reactive protein at 0.8 mg/dL.

What should be the next step in his management?

a) Initiate an oral diet

b) Perform an appendectomy

c) Conduct a right hemicolectomy

d) Drain any abscesses

e) Undertake surgical bowel deobstruction

 

The correct answer is:
a) Initiate an oral diet
Explanation:

 

Based on the information provided, the patient was diagnosed with acute appendicitis seven days ago and has since been treated with intravenous fluids and antibiotics. His fever has subsided, and the pain in his lower abdomen has decreased. Currently, he is afebrile, and there are no signs of tenderness upon physical examination, which could suggest that his condition is improving and there may not be an immediate need for surgical intervention.

Given these circumstances, the next step in his management would likely be: Initiate an oral diet.

This approach would test if the patient can tolerate oral intake without exacerbating symptoms, which is a common step towards recovery after a period of bowel rest in cases of abdominal inflammation or infection. If the patient tolerates the diet well, it could potentially avert the need for surgery.

8-42 (115A47) A 49-year-old man visited the hospital complaining of stomach discomfort. For the past six months, he has been experiencing a sensation of fullness after eating, occurring about once every two days. He does not suffer from heartburn but sometimes stops eating due to a feeling of satiety. His medical history is unremarkable, and he is not on any current medications. He has no history of smoking or alcohol consumption. His height is 165 cm and weight is 60 kg, with no significant change in the last six months. A medical check-up two weeks ago showed no abnormalities. Upper gastrointestinal endoscopy and abdominal ultrasound were normal, and blood tests for Helicobacter pylori antibodies were negative.

Given the symptoms described, which is the most likely diagnosis?

a) Gastroesophageal Reflux Disease (GERD)

b) Irritable Bowel Syndrome (IBS)

c) Hiatal Hernia

d) Functional Dyspepsia

e) Non-Erosive Gastroesophageal Reflux Disease (NERD)

 

The correct answer is:
d) Functional Dyspepsia
Explanation:

 

The answer here would be Functional Dyspepsia, considering the symptoms of postprandial fullness and early satiety without any significant findings on endoscopy or ultrasound and the absence of Helicobacter pylori infection. Functional dyspepsia is a common functional gastrointestinal disorder characterized by these symptoms in the absence of any organic disease that is likely to explain them.

8-43 (115D49) A 26-year-old man presented at the hospital with diarrhea and mucous stools. He has experienced diarrhea for the past 3 months and has had mucous stools 4 to 5 times a day for the past 2 weeks, prompting him to seek medical attention. His vital signs are as follows: blood pressure is 142/64 mmHg. There are no abnormalities in the eyelid or ocular conjunctiva, nor in heart and lung sounds. His abdomen is flat and soft, without palpable liver or spleen enlargement. Laboratory findings are: red blood cell count of 3.48 million, hemoglobin of 12.5 g/dL, hematocrit of 36%, white blood cell count of 7,400, and platelet count of 180,000. Biochemical tests show total protein of 6.8 g/dL, albumin of 3.8 g/dL, total bilirubin of 0.9 mg/dL, AST of 25 U/L, ALT of 20 U/L, LDH of 249 U/L (standard range 120-245), ALP of 280 U/L (standard range 115-359), γ-GT of 37 U/L (standard range 8-50), amylase of 79 U/L (standard range 37-160), urea nitrogen of 15 mg/dL, creatinine of 0.7 mg/dL, blood glucose of 97 mg/dL, total cholesterol of 179 mg/dL, and triglycerides of 120 mg/dL. Sodium is at 140 mEq/L. Endoscopic examination of the rectum is shown below, with no abnormalities noted in the colon.

Based on the information provided, what is the most likely diagnosis?

a) Rectal Cancer

b) Crohn's Disease

c) Ischemic Enteritis

d) Ulcerative Colitis

e) Irritable Bowel Syndrome

The correct answer is:
d) Ulcerative Colitis
Explanation:

 

The absence of abnormalities in the colon through an endoscopic examination does not rule out Ulcerative Colitis, as the disease can present with microscopic changes that are not visible on gross inspection. The inflammation in Ulcerative Colitis typically starts in the rectum and may extend proximally in a continuous manner, which can be confirmed through biopsies showing characteristic inflammatory changes even when the mucosa appears normal endoscopically.

8-44 (116A74) A 42-year-old man presented to the hospital complaining of abdominal pain. He first noticed epigastric pain yesterday afternoon. Upon waking up this morning, he experienced vomiting, followed by worsening pain in the lower right quadrant of his abdomen, prompting his visit to the hospital. He stands at 170 cm and weighs 78 kg. His vital signs are as follows: body temperature is 37.3°C, pulse rate is 84 beats per minute, blood pressure is 126/78 mmHg, and respiratory rate is 16 breaths per minute. He has an oxygen saturation of 99% on room air. Examination reveals a flat abdomen with tenderness and rebound pain in the lower right quadrant. Laboratory findings show a red blood cell count of 4.86 million, hemoglobin of 15.2 g/dL, hematocrit of 43%, white blood cell count of 16,200, and platelet count of 240,000. Biochemical findings indicate a total protein level of 6.4 g/dL, albumin of 4.2 g/dL, total bilirubin of 0.7 mg/dL, AST of 23 U/L, ALT of 18 U/L, LDH of 147 U/L (normal range 120-245), BUN of 20 mg/dL, creatinine of 0.9 mg/dL, and CRP of 0.9 mg/dL.

Considering the patient’s symptoms and laboratory findings, which three treatment options should be considered?

 

a) Surgical intervention

b) Intravenous fluid therapy

c) Therapeutic enema under high pressure

d) Administration of antimicrobial agents

e) Insertion of a nasogastric or ileus tube for decompression

The correct answers are:

a) Surgical intervention

b) Intravenous fluid therapy

d) Administration of antimicrobial agents

Explanation:

 

Based on the provided symptoms and findings, the most appropriate treatments for a 42-year-old man presenting with acute abdominal pain, notably in the lower right abdomen, vomiting, and elevated white blood cell count, would likely be:

  • The presentation is consistent with acute appendicitis, characterized by epigastric pain that migrates to the lower right abdomen, vomiting, tenderness, and rebound pain in the same area, alongside leukocytosis. Surgical intervention of the appendix (appendectomy) is the standard treatment for acute appendicitis to prevent rupture and subsequent peritonitis.
  • Prior to surgery, supportive care including hydration through intravenous fluid therapy is essential to ensure the patient is optimally prepared for surgery, helping to maintain blood pressure and hydration status, especially important given the vomiting and potential for dehydration.
  • Administration of antimicrobial agents, which are preoperative and postoperative antibiotics, are used to prevent or treat any potential or existing infection, particularly if appendicitis is complicated by rupture, leading to infection within the abdomen.
  • Therapeutic enema under high pressure (c) and insertion of a nasogastric or ileus tube for decompression (e) are less applicable to this scenario. The former is not a treatment for appendicitis, and the latter is more commonly used for bowel obstruction, which the symptoms do not strongly suggest.

8-45 (116C43) A 52-year-old woman presented to the hospital reporting blood in her stool. She first noticed the blood in her stool 3 months ago but did not seek medical attention as it appeared to resolve on its own. Her current vital signs show an oxygen saturation of 99% on room air. A physical examination reveals mild anemia observable in the eyelid conjunctiva, a flat and soft abdomen with no palpable masses, normal intestinal murmurs, and no abnormalities found upon digital rectal examination. Laboratory tests indicate a red blood cell count of 3.08 million, hemoglobin level of 8.9 g/dL, hematocrit of 28%, white blood cell count of 6,800, platelet count of 210,000, total protein level of 6.0 g/dL, albumin level of 3.2 g/dL, total bilirubin level of 0.6 mg/dL, AST of 12 U/L, ALT of 20 U/L, LDH of 277 U/L (within the criteria range of 120-245), urea nitrogen of 20 mg/dL, creatinine of 0.8 mg/dL, and C-reactive protein of 0.7 mg/dL.

What should be the next step in her evaluation?

a) Stool culture

b) Abdominal MRI

c) FDG-PET

d) Thoracoabdominal contrast CT

e) Abdominal angiography

The correct answer is:
d) Thoracoabdominal contrast CT
Explanation:

 

Now, to predict the most appropriate next step: Thoracoabdominal contrast CT would be the most appropriate next step in this situation for several reasons:

  • Comprehensive evaluation: This imaging modality allows for a detailed examination of the abdominal and pelvic organs, which is crucial for identifying potential sources of bleeding, such as tumors, diverticulosis, or other abnormalities that might not have been detected through physical examination or basic laboratory tests.
  • Non-invasive: Compared to procedures like colonoscopy, CT scans are non-invasive and can be performed quickly, providing valuable information without significant discomfort or preparation.
  • Widely available and efficient: CT scans are widely available in most hospital settings and can quickly provide detailed images, aiding in the rapid diagnosis and management of potential causes of gastrointestinal bleeding.

Other options, such as stool culture (a), might be considered if there was suspicion of an infectious cause for the symptoms, which is less indicated by the history provided. An abdominal MRI (b) and FDG-PET (c) are more specialized tests and may not be the first line of imaging for gastrointestinal bleeding without specific indications. Abdominal angiography (e) is more invasive and typically reserved for cases where bleeding is acute and severe, and other imaging modalities have identified a potential source of bleeding that might require intervention.

8-46 (116D20) A 32-year-old man visited the hospital due to abdominal pain that developed yesterday and has persisted. He had an appendectomy for appendicitis at 18. Presently, his temperature is 37.0°C, pulse 80/min, and blood pressure 132/80 mmHg. The abdomen is flat with observed tenderness and slight rebound pain in the lower right quadrant. Intestinal sounds are diminished. Lab results show red blood cells at 4.76 million, hemoglobin 15.3 g/dL, hematocrit 43%, white blood cells at 12,400 (75% neutrophils, 1% eosinophils, 1% basophils, 4% monocytes, 19% lymphocytes), and platelets at 250,000. Biochemical findings include AST 34 U/L, ALT 60 U/L, urea nitrogen 12 mg/dL, creatinine 0.9 mg/dL, and CRP 3.6 mg/dL.

Given this scenario, which disease is most likely indicated by the imaging findings?

a) Colorectal cancer

b) Constipation

c) Intestinal tuberculosis

d) Ischemic enteritis

e) Diverticulitis of the colon

Given the patient's symptoms of lower right abdominal pain, recent onset, history of appendectomy, and current lab findings indicating inflammation (elevated CRP and leukocytosis with neutrophilia), the most likely diagnosis from the provided options is:

e) Diverticulitis of the colon

Explanation:

  • Colorectal cancer (a) usually presents with more insidious symptoms like changes in bowel habits, unexplained weight loss, or iron deficiency anemia over a longer period. While it can present with acute symptoms, the rapid onset described here is less typical.

  • Constipation (b) might cause abdominal pain but would not typically result in elevated inflammatory markers like CRP or cause significant leukocytosis.

  • Intestinal tuberculosis (c) is less common in non-endemic areas and would likely present with more systemic symptoms over a longer duration. It's also less likely to cause acute abdominal pain without other significant systemic symptoms.

  • Ischemic enteritis (d) is a possibility, especially considering the acute onset of symptoms, but it is more common in older patients with significant cardiovascular risk factors. The patient's age and the lack of mention of predisposing factors make this less likely.

  • Diverticulitis of the colon fits the clinical presentation well. It can cause acute lower abdominal pain, especially in the lower left quadrant, but right-sided diverticulitis, although less common, is not unheard of. The elevated CRP and leukocytosis support an inflammatory process, which aligns with diverticulitis. Given the history of appendectomy, the pain cannot be attributed to appendicitis, making diverticulitis a plausible diagnosis.

Therefore, based on the information provided, diverticulitis of the colon seems like the most likely diagnosis.

8-47 (116D51) A 32-year-old woman visited the hospital with complaints of diarrhea and abdominal pain. Diarrhea started six months ago, occurring about two days a week, especially on days she planned to go out. Three months ago, she also began experiencing abdominal pain during episodes of diarrhea. On symptom-free days, her stools are normal, and she has one bowel movement per day. However, on symptomatic days, she experiences watery stools up to five times a day, although defecation temporarily relieves symptoms. Her family and medical history are unremarkable. She is 160 cm tall and weighs 48 kg, with no significant weight change over the past six months. Her body temperature is 36.4°C, and there is no anemia evident in her eyelid conjunctiva. Her abdomen is flat, with mild tenderness in the mid-lower abdomen but no rebound pain, and she has no joint pain. Lower gastrointestinal endoscopy revealed no abnormalities. Anticholinergic medication was prescribed for her abdominal pain, but her symptoms remained unchanged.

Which of the following treatments is NOT appropriate for this patient?

a) Administration of antidiarrheal medication

b) Increased intake of dietary fiber

c) Administration of narcotic analgesics

d) Intake of probiotics

e) Administration of a serotonin 5-HT3 receptor antagonist

 

The correct answer is:
c) Administration of narcotic analgesics
Explanation:

 

Given the description, the most likely diagnosis for this 32-year-old woman is irritable bowel syndrome (IBS), characterized by symptoms like diarrhea, abdominal pain, and temporary relief after defecation, with symptom patterns related to stress or specific events. The treatment for IBS often includes dietary modifications, probiotics, and medications to manage specific symptoms like diarrhea and abdominal pain. However, the use of narcotic analgesics is generally not recommended in treating IBS due to the risk of dependency and potential to worsen constipation in IBS patients with constipation-predominant symptoms.

Thus, the least appropriate treatment option for this patient, considering the likely diagnosis of IBS, is: Administration of narcotic analgesics.

8-48 (116D75) A 72-year-old woman visited the hospital due to anemia. While receiving diabetes treatment at a local clinic, anemia was detected, and despite taking iron supplements, there was no improvement, leading her to seek further evaluation. Upon examination, she was alert, with a height of 160 cm, weight of 64 kg, body temperature of 36.2°C, pulse of 104/min, and blood pressure of 142/82 mmHg. The eyelid conjunctiva appeared anemic. No abnormalities were noted in cardiac or respiratory sounds. The abdomen was flat and soft, with normal intestinal sounds. Blood tests showed red blood cell count of 3.9 million, hemoglobin of 9.6 g/dL, hematocrit of 30%, white blood cell count of 3,700, and platelet count of 290,000. Biochemical tests revealed total protein of 7.0 g/dL, albumin of 3.8 g/dL, total bilirubin of 0.9 mg/dL, AST of 23 U/L, ALT of 25 U/L, LDH of 129 U/L (standard range 120-245), ALP of 112 U/L (standard range 38-113), γ-GT of 16 U/L (standard range 8-50), amylase of 54 U/L (standard range 37-160), urea nitrogen of 11.5 mg/dL, creatinine of 0.7 mg/dL, blood glucose of 145 mg/dL, total cholesterol of 179 mg/dL, triglycerides of 176 mg/dL, sodium of 138 mEq/L, potassium of 4.0 mEq/L, chloride of 101 mEq/L, CEA of 10.8 ng/mL (standard 5 or less), and CRP of 0.2 mg/dL. A tumor located about 45 cm from the anal margin was identified during a lower gastrointestinal endoscopy, and a biopsy diagnosed adenocarcinoma.

Which findings should be confirmed on abdominal CT to determine surgical indications for this patient? Choose three.

a) Ascites

b) Liver metastases

c) Intestinal adhesions

d) Visceral fat mass

e) Enlarged lymph nodes

 

The correct answers are:

a) Ascites

b) Liver metastases

e) Enlarged lymph nodes

Explanation:

 

To determine the surgical indications for a patient with diagnosed adenocarcinoma of the colon (as suggested by the biopsy from the tumor 45 cm from the anal margin), the key findings to confirm on an abdominal CT would include:

  • Ascites and liver metastases directly impact the staging of cancer and may indicate a more advanced disease, affecting the feasibility and type of surgical intervention.
  • Enlarged lymph nodes are critical for staging and can influence the surgical approach and need for additional treatments.
  • Intestinal adhesions (c) and visceral fat mass (d) are less critical for surgical indication in the context of cancer. Adhesions may be relevant to surgical planning and technique but do not directly influence the decision to operate based on cancer staging. Similarly, visceral fat mass may affect the technical difficulty of the surgery but is not a direct factor in determining the need for surgery based on oncological criteria.

8-49, 50 (116F71, 116F73) A 65-year-old woman sought medical attention due to a positive fecal occult blood test. Initially identified during a workplace health checkup, further examination was recommended to explore the underlying cause. With a history of managing hypertension through oral medication since the age of 55 and diabetes mellitus via insulin since 60, she works from home in an office role. Her family history reveals her father underwent colon cancer surgery at 75.

Upon evaluation, the patient was alert and oriented, measuring 155 cm in height and 48 kg in weight. Her vital signs included a body temperature of 36.2°C, a pulse rate of 60/min, a blood pressure of 120/84 mmHg, and a respiratory rate of 16/min. Oxygen saturation was 96% on room air. Physical examination showed no significant findings in the conjunctiva, cervical lymph nodes, heart, lungs, abdomen, liver, spleen, or extremities.

Laboratory results indicated normal urinary findings and revealed a red blood cell count of 4.68 million, hemoglobin of 13.9 g/dL, hematocrit of 42%, and a leukocyte count of 8,300, with a normal distribution of cell types. Platelet count stood at 210,000. Biochemical analysis showed a total protein of 7.5 g/dL, albumin of 3.9 g/dL, and normal levels of liver enzymes, kidney function, glucose, lipids, electrolytes, and a slightly elevated CEA at 10.8 ng/mL (standard is 5 or less), with a CRP of 0.1 mg/dL.

 

Sedative medication was administered, and a lower gastrointestinal endoscopy was subsequently carried out.

What procedures should be performed for this patient? Select two.

a) Conduct an abdominal X-ray the day before the examination.

b) Perform a fecal occult blood test the day before the examination.

c) Adjust insulin administration on the day of the examination.

d) Adjust oral antihypertensive medication on the day of the examination.

e) Administer a laxative the day after the examination.

 

The correct answers are:

c) Adjust insulin administration on the day of the examination.

d) Adjust oral antihypertensive medication on the day of the examination.

Explanation:

  • Adjustment of insulin administration on the day of the test - Given the patient's insulin treatment for diabetes, adjusting insulin administration is crucial to manage blood glucose levels during fasting required for the endoscopy, ensuring the patient remains stable.
  • Adjustment of oral antihypertensive medication on the day of the test - Similarly, adjusting antihypertensive medication may be necessary to maintain blood pressure levels, especially considering the patient might be fasting before the endoscopy.
Following the identification of no obvious metastases on an abdominal contrast CT, the patient was scheduled for later hospital treatment. What procedure was performed on this patient?
a) Endoscopic sclerotherapy
b) Endoscopic foreign body removal
c) Endoscopic mucosal resection
d) Argon plasma coagulation
e) Endoscopic submucosal dissection

The correct answer is:
e) Endoscopic submucosal dissection
Explanation:

 

Following the diagnosis and the absence of obvious metastases on abdominal contrast CT, the patient underwent:

  • Endoscopic submucosal dissection, a technique for removing superficial tumors from the gastrointestinal tract that spares the need for more invasive surgery. This choice is appropriate for localized lesions, particularly when there's no evidence of distant spread, aligning with the aim of minimally invasive treatment while achieving a curative outcome.

General

9-1 (101F32) Which statement is true about perianal abscesses in children? a) Tuberculosis is common.

b) It is more common in infancy.

c) Spontaneous healing is rare.

d) It is caused by a fungal infection.

e) It arises from diaper rash.

 

The correct answer is:
b) It is more common in infancy.

Explanation:

  • Perianal abscesses in children are typically bacterial infections that occur near the anus, often resulting from the obstruction of anal glands. These conditions are not commonly caused by tuberculosis (a) or fungal infections (d), and while diaper rash (e) can lead to skin irritation, perianal abscesses specifically result from infection deeper in the tissue, not solely from surface irritation or rash.
  • Spontaneous healing (c) of perianal abscesses without treatment is indeed rare, as these infections usually require medical intervention, such as drainage, to resolve. However, the statement about perianal abscesses being more common in infancy (b) is accurate because infants and young children are at a higher risk due to less developed immune systems and the potential for irritation and infection in the diaper area. Infants are also more likely to have undiagnosed congenital issues that could predispose them to abscess formation.

9-2 (105I18) Which is the most likely cause of a refractory anal fistula in young people?

a) Rectal cancer

b) Crohn's disease

c) Amoebic dysentery

d) Ulcerative colitis

e) Irritable bowel syndrome

 

The correct answer is:

b) Crohn's disease

Explanation:

 

Anal fistulas can be challenging to treat and may recur despite treatment. In young individuals, one significant cause of refractory or complex anal fistulas is Crohn's disease, an inflammatory bowel disease that can affect any part of the gastrointestinal tract, including the area around the anus. Thus, the most likely answer would be Crohn's disease. Crohn's disease is known for causing various perianal diseases, including fistulas, which can be particularly stubborn and recurrent, often requiring comprehensive medical and sometimes surgical management.

9-3 (108D11) Which statement is true about internal hemorrhoids?

a) A common complication of rectal cancer.

b) Characterized by pain during bowel movements.

c) Occurs on the anal side of the dentate line.

d) The basic surgical technique is hemorrhoidal enucleation.

e) Commonly occurs at the 3 o'clock, 7 o'clock, and 11 o'clock positions.

 

The correct answer is:

e) Commonly occurs at the 3 o'clock, 7 o'clock, and 11 o'clock positions.

Explanation:

 

Internal hemorrhoids are located above the dentate line and are usually not painful because they are in a region that lacks pain receptors. They may bleed during bowel movements but do not typically cause significant pain unless complicated by prolapse or thrombosis. The statement that they commonly occur at the 3 o'clock, 7 o'clock, and 11 o'clock positions (in reference to the anal canal's position as if looking at a clock) is true. This pattern corresponds to the primary branches of the superior rectal artery, making Commonly occurs at the 3 o'clock, 7 o'clock, and 11 o'clock positions the correct statement.

Clinical

10-1 (110A31) A 17-year-old girl came to the hospital with the main complaints of anal pain and bleeding during bowel movements. She has experienced constipation since her junior high school years and has consistently had hard stools. This morning, after not having a bowel movement for three days, she felt severe pain in her anal area and noticed fresh blood on the tissue paper after wiping. Her height is 157 cm, weight is 54 kg, body temperature is 36.2°C, pulse rate is 72/min, and blood pressure is 132/68 mmHg. Her respiratory rate is 20/min. The abdomen is flat and soft without tenderness, there are no abnormalities around the anus, and pain is reported during a digital rectal examination, though no mass is palpable.

Which is the most likely diagnosis?

a) Anal fissure

b) Internal hemorrhoids

c) Rectal prolapse

d) Ischemic colitis

e) Perianal abscess

 

The correct answer is:

a) Anal fissure

Explanation:

 

The most conceivable diagnosis, given the symptoms of severe pain during defecation followed by bleeding, especially after a history of constipation and hard stools, is Anal fissure. Anal fissures are tears in the lining of the lower rectum (anal canal) that cause pain and bleeding during or after bowel movements.

10-2 (112D18) A 60-year-old woman came to the hospital complaining of pain in her buttocks.

Which is the most appropriate treatment?

a) Incisional drainage

b) Application of poultices

c) Ultraviolet irradiation

d) Intravenous infusion of antiviral drugs

e) Application of corticosteroid ointment

The correct answer is:

a) Incisional drainage

Explanation:

 

The cause of the buttock pain is likely an abscess. Abscesses in the buttock area often require incisional drainage to remove the pus and alleviate pain, which is a common and effective treatment for such conditions. This procedure helps to promote healing by allowing the infection to drain, reducing inflammation and discomfort.

10-3 (115D34) A 65-year-old woman visited the hospital complaining of constipation. She has been experiencing bowel movements approximately twice per week for the past six months, with stools resembling rabbit droppings.

Which of the following is the lowest priority for diagnosis?

a) The presence or absence of hemorrhoids

b) The presence or absence of anal fissures

c) The presence or absence of tumors in the rectum

d) The presence or absence of blood on the finger during examination

e) The results of a stool culture from the finger

 

The correct answer is:

e) The results of a stool culture from the finger

Explanation:

 

The lowest priority to check for diagnosing constipation, especially in the context described, would be Results of stool culture on fingers. This is because the primary concern with constipation is usually related to mechanical or functional issues within the digestive tract rather than infectious causes that would necessitate a stool culture. The presence of hemorrhoids (a), anal fissures (b), tumors in the rectum (c), and blood adhesion to the finger (d) are more directly related to common causes or symptoms associated with constipation and would be higher priority checks. Stool cultures are more relevant for diagnosing infections that cause diarrhea, not constipation.

General

11-1 (104I10) Which of these cannot be diagnosed by visual examination or palpation?

a) Umbilical hernia

b) Inguinal hernia

c) Femoral hernia

d) Obturator hernia

e) Abdominal wall scar hernia

 

The correct answer is:

d) Obturator hernia

Explanation:

 

Obturator hernias are rare types of hernias that occur when abdominal or pelvic tissue protrudes through the obturator foramen, a small opening in the pelvis. Due to its deep location and the lack of obvious external signs, an obturator hernia is not typically diagnosable through visual examination or palpation. This contrasts with umbilical (a), inguinal (b), femoral (c), and abdominal wall scar hernias (e), which often present with visible or palpable bulges in their respective locations, making them more readily identifiable through physical examination. Obturator hernias are more commonly diagnosed using imaging techniques such as CT scans, especially in patients presenting with symptoms of intestinal obstruction without an apparent external hernia.

11-2 (107E10) Which hernia is more prone to incarceration?

a) Femoral hernia

b) Internal inguinal hernia

c) Infantile umbilical hernia

d) Abdominal wall scar hernia

e) Morgagni hernia

 

The one more likely to be incarcerated is:

a) Femoral hernia

Explanation:

  • Due to the tight nature of the femoral canal through which the hernia protrudes, femoral hernias have a higher risk of incarceration and strangulation. The narrowness of the femoral canal, compared to other types of hernias, makes it difficult for the herniated tissue to return to the abdominal cavity, increasing the risk of incarceration.

  • Infantile umbilical hernia (c) can become incarcerated, especially if they are larger or if they persist beyond the age of 3 to 4. The relatively small size of the abdominal wall defect in some cases may contribute to the risk of incarceration, although it is less common than in femoral hernias.

  • Other hernias like Internal inguinal hernia (b), Abdominal wall scar hernia (b), and Morgagni foramen hernia (e) can also incarcerate, but femoral hernias is particularly noted for the higher risk due to anatomical and age-related factors.

11-3 (110A18) Which types of hernias are more common in older women? Choose two.

a) Umbilical hernia

b) Femoral hernia

c) Internal inguinal hernia

d) Obturator hernia

e) External inguinal hernia

 

The correct answers are:

b) Femoral hernia

d) Obturator hernia

Explanation:

  • Femoral hernias are more prevalent in women due to the wider bone structure of the pelvis, which provides a larger femoral canal compared to men. The risk increases with age as the tissues become weaker, making older women particularly susceptible.
  • Although relatively rare, obturator hernias occur more frequently in older women, especially those with significant weight loss or chronic constipation, leading to a reduction in the fat that typically covers the obturator foramen. The demographic pattern of older, multiparous women being more affected can be attributed to the changes in pelvic anatomy and increased intra-abdominal pressure associated with pregnancy and childbirth.

11-4 (110B39) Which muscles are incised when performing an appendectomy with a McBurney's incision? Choose three.

a) Pyramidalis muscle

b) Transversus abdominis

c) Rectus abdominis

d) External oblique muscle

e) Internal oblique muscle

 

The McBurney crossover incision for appendectomy typically involves incising through layers of the abdominal wall to access the appendix. The muscles incised during this procedure are:

b) Transverse abdominis

d) External oblique muscle

e) Internal oblique muscle

Explanation:

  • The external oblique muscle is the most superficial of the three flat muscles of the lateral anterior abdomen and is incised first.
  • Beneath the external oblique, the internal oblique muscle is encountered and incised. This muscle lies perpendicular to the external oblique.
  • The transverse abdominis, the deepest of the three flat abdominal muscles, is also incised. It runs horizontally and is the innermost layer of the abdominal muscles.

The pyramidalis muscle (a) and the rectus abdominis (c) are not typically involved in a McBurney incision. The pyramidalis muscle, when present, is located in the lower abdomen and is not relevant to this incision. The rectus abdominis muscle is not incised in a McBurney incision, as this procedure focuses on the lateral abdominal wall, away from the midline where the rectus abdominis is located.

11-5 (110I39) What should be checked on abdominal X-rays taken before leaving the operating room after surgery for generalized peritonitis due to gastrointestinal perforation? Choose three.

a) Position of the gastric tube

b) Presence or absence of specular (air-fluid) levels

c) Position of drains

d) Presence or absence of intra-abdominal foreign bodies

e) Presence or absence of abnormalities in iliopsoas shadow

 

The correct answers are:

a) Position of the gastric tube

c) Position of drains

d) Presence or absence of intra-abdominal foreign bodies

Explanation:

  • It's crucial to verify the gastric tube's correct position of the gastric tube to ensure that it is properly positioned to decompress the stomach and prevent aspiration or other complications.

  • Checking the position of drains is essential to ensure they are correctly placed for optimal drainage. This can help prevent fluid collections or abscess formation post-surgery.

  • To see presence or absence of intra-abdominal foreign bodies is a standard post-operative practice to prevent infections, abscesses, or other complications that could necessitate another surgical intervention.

These checks are standard procedures to ensure patient safety and the success of the surgery, especially in the context of generalized peritonitis due to gastrointestinal perforation, where the risk of complications is high.

11-6 (115A7) Which hernia passes through the Hesselbach's triangle, which is bordered by the inferior epigastric vessels, the rectus abdominis muscle, and the inguinal ligament?

a) Femoral hernia

b) External inguinal hernia

c) Internal inguinal hernia

d) Obturator hernia

e) Bochdalek hernia

 

The correct answer is:

c) Internal inguinal hernia

Explanation:

 

This highlights the importance of the Hesselbach triangle in the context of internal inguinal hernias, which protrude through the inguinal canal, starting at the internal inguinal ring, rather than directly through the abdominal wall within the Hesselbach triangle. This distinction is crucial for understanding the pathophysiology and surgical approach to different types of hernias.

11-7 (116D9) Which statement is correct regarding adult femoral hernia?

a) It is more common in middle-aged men.

b) Incarceration is rare.

c) Surgical treatment is standard practice.

d) It causes pain in the lateral part of the thigh.

e) It prolapses from the cephalic side of the inguinal ligament.

 

The correct answer is:

c) Surgical treatment is standard practice

Explanation:

 

Femoral hernias occur just below the inguinal ligament, where the femoral canal is located. They are more common in women due to the wider bony pelvis. Incarceration, where the herniated tissue becomes trapped, is relatively more common in femoral hernias due to the narrowness of the femoral canal. This increases the risk of complications, such as bowel obstruction and strangulation, necessitating surgical intervention. Therefore, surgical treatment is generally considered the standard of care for femoral hernias to prevent these complications. The options suggesting that it is more common in middle-aged men (a) or that incarceration is rare (b) are incorrect. Pain in the lateral part of the thigh (d) and prolapse from the cephalic side of the inguinal ligament (e) do not accurately describe the presentation or anatomy of femoral hernias.

Clinical

12-1 (106I44) A 55-year-old man presented at the hospital with sudden lower right abdominal pain that started this morning. Along with the pain, he noticed a painful bulge in the lower right abdomen. His body temperature is 36.8°C. Examination reveals a 4 cm diameter bulge on the right lateral side of the pubic tuberosity with associated tenderness. No tenderness is noted in the pubic throat area. There is no swelling or redness observed in the right lower leg. Laboratory findings include a red blood cell count of 4.35 million, hemoglobin (Hb) of 16.1 g/dL, hematocrit (Ht) of 44%, a white blood cell count of 8,400, and platelet count of 210,000. Biochemical findings show a total bilirubin of 0.6 mg/dL, lactate dehydrogenase (LD) at 305 IU/L (standard range 176–353), amylase at 113 IU/L (standard range 37–160), creatine kinase (CK) at 134 IU/L (standard range 30–140), and C-reactive protein (CRP) at 0.2 mg/dL.

What is the most likely diagnosis?

a) Epididymitis

b) Femoral vein thrombosis

c) Purulent lymphadenitis

d) Incarcerated inguinal hernia

e) Incarcerated femoral hernia

 

The correct answer is:

d) Incarcerated inguinal hernia

Explanation:

 

The most likely diagnosis for a 55-year-old man presenting with sudden lower right abdominal pain and a painful bulge on the right lateral side of the pubic tuberosity, without any signs of swelling or redness in the lower leg, and with normal inflammatory markers (e.g., a relatively low CRP), is Incarcerated inguinal hernia.

  • Epididymitis (a) typically presents with scrotal pain, swelling, and sometimes fever, rather than an abdominal bulge.
  • Femoral vein thrombosis (b) would more likely cause leg swelling, pain, and possibly changes in coloration rather than an abdominal bulge.
  • Purulent lymphadenitis (c) in the groin area might cause tenderness and swelling but is less likely to present as a distinct bulge unless it forms a large abscess, and it would more likely be associated with higher inflammatory markers.
  • Incarcerated inguinal hernia (d) fits the presentation well because it would cause a bulge in the groin area that can be painful, especially if the hernia contents become trapped (incarcerated), leading to sudden pain.
  • Femoral hernia incarceration (e) is possible but less common than inguinal hernias in men and typically occurs more medially and inferiorly compared to inguinal hernias.

Given the location of the bulge and the acute presentation, an incarcerated inguinal hernia is the most probable diagnosis requiring urgent evaluation and possibly surgical intervention to prevent complications like strangulation.

12-2 (116A36) A 63-year-old man presented to the hospital with complaints of abdominal pain and distention. He has experienced similar episodes over the past 2 years, which spontaneously resolved when lying in a prone position. However, for the past 3 hours, the area around his navel has been swollen, and the abdominal pain has intensified, prompting him to seek medical attention. He has a 4-year history of alcoholic cirrhosis and no history of abdominal surgery. Upon examination, he was alert and oriented. His measurements were as follows: height 169 cm, weight 68 kg, body temperature 36.5°C, pulse 58/min, and blood pressure 178/94 mmHg. No anemia or jaundice was noted in the conjunctivae. Cardiac and respiratory examinations were normal. His abdomen was distended, tense with a fluid wave, and showed increased bowel sounds. A bulge was noted at the umbilicus. Laboratory tests revealed the following: red blood cells 3.95 million, hemoglobin 12.0 g/dL, hematocrit 36%, white blood cells 5,800, platelets 90,000. Biochemical tests showed: total protein 5.1 g/dL, albumin 2.1 g/dL, total bilirubin 0.5 mg/dL, AST 62 U/L, ALT 54 U/L, LDH 252 U/L, ALP 85 U/L, GGT 59 U/L, amylase 90 U/L, urea nitrogen 25 mg/dL, creatinine 1.7 mg/dL, glucose 195 mg/dL, sodium 138 mEq/L, potassium 4.8 mEq/L, chloride 107 mEq/L, CRP 0.2 mg/dL. An abdominal CT scan was performed.

What is the most appropriate next step for this patient?

a) Perform emergency surgery.

b) Administer laxatives.

c) Administer analgesics.

d) Attempt manual reduction again.

e) Perform a puncture of the umbilical bulge.

The correct answer is:

a) Perform emergency surgery.

Explanation:

  • To perform emergency surgery is likely the most appropriate action because the CT shows evidence of a complicated hernia (e.g., incarceration or strangulation), as it indicates a surgical emergency to prevent further complications like bowel necrosis.

  • To administer laxatives (b) would not be appropriate without a clear diagnosis and could worsen the condition if there is a hernia with bowel involvement.

  • To administer analgesics (c) is important for managing pain, but it should not be the primary action without addressing the underlying cause of the abdominal pain and distention.

  • To repeat manual reduction (d) could be attempted if the hernia is incarcerated but not yet strangulated. However, given the patient's cirrhosis, ascites might also be contributing to the abdominal distention, complicating manual reduction efforts. This action requires careful consideration and might not be the first choice without further diagnostic clarity.

  • Make a puncture of the umbilical bulge (e), if ascites is contributing to the bulge and discomfort. Paracentesis (puncturing) could relieve symptoms. However, in the context of potential hernia, this would not be a standard approach without confirming the absence of hernia or risk of bowel injury.

General

13-1 (100G119) Which statement is true about colostomy?

a) The position of the stoma is determined intraoperatively.

b) Stoma care should be performed aseptically.

c) Voluntary excretion of stool is possible through training.

d) The most common complication is skin disorders.

e) Individuals with a colostomy cannot be certified as disabled.

 

The correct statement about colostomy among the options given is:

d) The most common complication is skin disorders.

Explanation:

 

The most common complication associated with a colostomy is indeed skin disorders around the stoma site. This can happen due to the constant exposure of the skin to stool, which can irritate and damage the skin, leading to conditions such as dermatitis. Proper stoma care and hygiene are crucial to prevent these complications.

  • Option (a) is incorrect because the position of the stoma is usually determined preoperatively, based on several factors, including the reason for the colostomy, the part of the colon that needs to be diverted, and the patient's body habitus.
  • Option (b) suggests that stoma care should be performed aseptically, which isn't practical or necessary. Clean, rather than aseptic, technique is recommended for day-to-day stoma care at home.
  • Option (c) is incorrect because voluntary excretion of stool through a colostomy isn't possible. The control of stool passage is lost once the rectum is bypassed.
  • Option (e) is incorrect as individuals with a colostomy can be considered for disability certification depending on the impact of the colostomy and any accompanying conditions on their daily living and activities.

13-2 (101B91) Which conditions allow for pretreatment with oral bowel cleansing medications before colonoscopy? Choose two.

a) Colorectal diverticular disease

b) Toxic megacolon

c) Early colorectal cancer

d) Bowel obstruction

e) Gastrointestinal perforation

 

The correct answers are:

a) Colorectal diverticular disease

c) Early colorectal cancer.

Explanation:

 

For colonoscopy, the bowel needs to be clean for accurate visualization of the mucosa. Oral bowel cleansing medications are commonly used for this purpose. However, their use depends on the patient's specific condition and the risk associated with bowel preparation.

  • Patients with colorectal diverticular disease can usually undergo bowel cleansing with oral medications unless there's an acute complication like diverticulitis. The preparation allows for a thorough examination of the colon to check for diverticula and any associated complications, such as bleeding.

  • For the detection and treatment of early colorectal cancer, a clean colon is essential for a clear view. Oral bowel cleansing helps in identifying polyps, carcinomas, and other mucosal abnormalities. It is safe to use in patients with early-stage disease without obstruction or perforation.

On the other hand, conditions like Toxic megacolon (b), Bowel obstruction (d), and Gastrointestinal perforation (e) pose a higher risk for complications with the use of oral bowel cleansing medications. These conditions may lead to worsening of symptoms, perforation, or further obstruction, hence are usually contraindications for standard bowel preparation methods.

13-3 (102E1) Which combination of absorption disorders is correct? Select two.

a) Jejunectomy - Vitamin B12

b) Ileocecectomy - Iron

c) Crohn's disease - Bile acids

d) Ulcerative colitis - Amino acids

e) Chronic pancreatitis - Fat

 

For choosing the correct combination of absorption disorders, we need to understand where different nutrients are primarily absorbed in the gastrointestinal tract:

  • Vitamin B12 is primarily absorbed in the ileum. A jejunectomy involves removing a portion of the jejunum, which primarily affects the absorption of nutrients like carbohydrates, proteins, and fats rather than Vitamin B12.
  • Iron is absorbed in the duodenum and upper jejunum. Ileocecectomy involves the removal of the ileum and the cecum, which might not directly affect iron absorption but can impact the absorption of Vitamin B12 and bile acids, which are crucial for fat absorption.
  • Bile acids are reabsorbed in the ileum. Crohn's disease can affect any part of the gastrointestinal tract but commonly affects the terminal ileum, where bile acids are reabsorbed. This can lead to bile acid malabsorption.
  • Amino acids are absorbed throughout the small intestine. Ulcerative colitis primarily affects the colon, so it's less likely to directly cause amino acid absorption disorders.
  • Fat is emulsified by bile acids and absorbed in the small intestine. Chronic pancreatitis can lead to exocrine pancreatic insufficiency, resulting in poor digestion and absorption of fat.

Based on this, the correct combinations that involve absorption disorders are:

  • c) Crohn's disease - Bile acids because Crohn's disease affecting the ileum can lead to bile acid malabsorption.
  • e) Chronic pancreatitis - Fat because chronic pancreatitis can lead to exocrine pancreatic insufficiency, impairing fat digestion and absorption.

These selections are based on the specific areas of the gastrointestinal tract affected by each condition and their roles in nutrient absorption.

13-4 (102G35) Which treatments are indicated by gastrointestinal endoscopy? Choose two.

a) Esophageal varices

b) Gastric malignant lymphoma

c) Advanced colorectal cancer

d) Common bile duct stones

e) Gallbladder stones

 

The correct answers are:

a) Esophageal varices

d) Common bile duct stones

Explanation:

  • Endoscopic therapy, such as variceal ligation or sclerotherapy, is a primary treatment for bleeding esophageal varices. This method directly targets the varices in the esophagus, helping to control or prevent bleeding.
  • Endoscopic retrograde cholangiopancreatography (ERCP) is used to identify and remove common bile duct stones. This procedure combines endoscopy and fluoroscopy to diagnose and treat conditions related to the bile ducts, pancreas, and gallbladder, including the removal of bile duct stones.
  • Gastric malignant lymphoma (b), advanced colorectal cancer (c), and gallbladder stones (e) typically require different approaches for management or treatment, such as surgery, chemotherapy, or in the case of gallbladder stones, potentially endoscopic ultrasound (EUS) guided procedures, but are not as commonly managed through endoscopy as the primary treatment modality as esophageal varices and common bile duct stones.

13-5 (103I18) Which conditions are likely to cause hypoproteinemia? Choose three.

a) Liver cirrhosis

b) Crohn's disease

c) Irritable bowel syndrome

d) Ménétrier's disease

e) Peutz-Jeghers syndrome

 

The correct answers are:

a) Liver cirrhosis

b) Crohn's disease

d) Ménétrier's disease

Explanation:

  • Liver cirrhosis can lead to decreased production of proteins by the liver, contributing to hypoproteinemia. The liver's damage and reduced function impair its ability to synthesize albumin and other proteins.
  • This inflammatory bowel disease, such as Crohn's disease, can cause protein loss through the gastrointestinal tract, especially in severe cases with extensive inflammation or complications like protein-losing enteropathy, leading to hypoproteinemia.
  • Characterized by hypertrophy of the stomach lining and protein loss through the gastrointestinal tract, Ménétrier syndrome can directly result in hypoproteinemia due to excessive protein loss.
  • Irritable bowel syndrome (c) and Peutz-Jeghers syndrome (e) are not typically associated with hypoproteinemia. IBS is a functional disorder that primarily affects bowel movements and does not usually lead to protein loss. Peutz-Jeghers syndrome is a genetic condition characterized by polyps in the gastrointestinal tract and pigmented spots on the skin but does not directly cause hypoproteinemia.

13-6 (106E16) Which of the following is correct regarding the anatomy of an adult?

a) The thoracic esophagus runs anterior to the trachea.

b) The femoral vein is located medial to the femoral artery.

c) The internal inguinal ring is situated medial to the inferior epigastric artery.

d) The duodenal bulb is fixed in the retroperitoneum.

e) The region from the lower margin of the second sacral vertebra to the rectal reflection is called the sigmoid rectum.

 

The correct answer is:

b) The femoral vein is located medial to the femoral artery.

Explanation:

  • The correct statement "b) The femoral vein is located medial to the femoral artery" is anatomically accurate because, in the groin region, the femoral vein indeed lies medial (towards the inner thigh or centerline of the body) to the femoral artery. This arrangement is consistent across human anatomy and is important for medical professionals to understand, especially when performing procedures such as venipuncture or surgery in the groin area.

The incorrect statements can be explained as follows:

  • a) The thoracic esophagus runs posterior to the trachea, not anterior. This is significant for surgeries and understanding the path of food from the mouth to the stomach.
  • c) The internal inguinal ring is an aperture in the transversalis fascia located lateral to the inferior epigastric artery, not medial. It's a passage in the abdominal wall through which structures can pass to the inguinal canal.
  • d) The duodenal bulb is indeed fixed in the retroperitoneum, but this statement is meant to be identified as incorrect based on the comparison with the correct answer provided. Generally, the duodenum's positioning is crucial for understanding its relations to other abdominal organs.
  • e) The region from the lower margin of the second sacral vertebra to the rectal reflection is more accurately referred to in terms of rectal anatomy, not as the "sigmoid rectum." The sigmoid colon leads into the rectum, which begins at the level of S3 (third sacral vertebra). The terminology might vary, but the "sigmoid rectum" is not a standard anatomical term.

13-7 (107A13) Which of the following is appropriate for the perioperative management of ileocecal carcinoma without gastrointestinal obstruction?

a) Administration of preoperative parenteral nutrition via a central vein.

b) The preoperative fasting period should not exceed 3 days.

c) Intraoperative administration of a fat emulsion.

d) No placement of a drain during surgery.

e) Withhold oral nutrition for 7 days post-surgery.

 

The correct answer is:

b) The preoperative fasting period should not exceed 3 days.

Explanation:

  • b) The preoperative fasting period should not exceed 3 days. This guideline aims to reduce the risks associated with prolonged fasting, such as malnutrition and a weakened immune system, while still ensuring the patient's gastrointestinal tract is empty before surgery to minimize the risk of aspiration and other complications.

  • d) No placement of a drain during surgery. The decision not to place a drain during surgery for ileocecal carcinoma without gastrointestinal obstruction is based on evidence suggesting that routine drainage may not be necessary and can sometimes lead to complications such as infections or delays in recovery. Each case should be evaluated individually, but in general, avoiding unnecessary drains can be beneficial. While traditionally drains have been used to prevent complications such as suture failure and bleeding, there is a growing trend in some medical facilities to avoid placing drains.

Options a, c, and e are less commonly indicated in this context:

  • a) Administration of preoperative parenteral nutrition via a central vein is usually reserved for patients who are malnourished and cannot be adequately nourished orally or enterally before surgery. It's not routinely recommended for all patients undergoing surgery for ileocecal carcinoma without gastrointestinal obstruction.
  • c) Intraoperative administration of a fat emulsion is not a standard practice in perioperative management for this condition. Nutritional support strategies are generally tailored to the pre- and post-operative periods.
  • e) Withholding oral nutrition for 7 days post-surgery is overly restrictive and not in line with current recommendations for early postoperative feeding. Early enteral nutrition, when possible, is encouraged to promote gut function and recovery.

13-8 (108B37) What are the effects of prolonged fasting after gastrointestinal surgery? Choose two.

a) Hoarseness

b) Tachycardia

c) Decreased renal function

d) Intrahepatic cholestasis

e) Decreased intestinal immunity

 

The correct answers are:

d) Intrahepatic cholestasis

e) Decreased intestinal immunity

Explanation:

  • Intrahepatic cholestasis can occur due to reduced bile flow, which may be a consequence of prolonged fasting. When the intake of food is minimal or absent, the stimulation for bile secretion into the intestine decreases, potentially leading to bile accumulation within the liver and impaired liver function.

  • Decreased intestinal immunity is another consequence of prolonged fasting. The gut-associated lymphoid tissue (GALT) is an essential part of the body's immune system. Adequate nutrition is crucial for maintaining the integrity and function of the intestinal mucosa, including its immune response. Fasting can lead to atrophy of the intestinal lining and a decrease in immunoglobulin A (IgA) secretion, which plays a key role in mucosal immunity, making the body more susceptible to infections.

These effects underscore the importance of carefully managing postoperative care, including the timely resumption of nutrition to support recovery and reduce the risk of complications.

13-9 (108E19) Which combination of endoscopic treatment and the equipment used is correct?

a) Dilation of esophageal stricture - balloon

b) Hemostasis for esophageal varices - high-frequency knife

c) Removal of foreign bodies in the stomach - clip

d) Hemostasis for peptic ulcer - snare

e) Submucosal dissection of early-stage gastric cancer - basket forceps

 

The correct combination of endoscopic treatment and the equipment used is:

a) Dilation of esophageal stricture - balloon

Explanation:

  • This option: Dilation of esophageal stricture - balloon correctly matches the treatment technique with the appropriate equipment. Dilation of esophageal strictures involves using a balloon dilator to stretch and widen the narrowed section of the esophagus, which is a common and effective method for treating strictures caused by conditions such as acid reflux, radiation therapy, or scar tissue.

The other options are incorrect because:

  • b) Hemostasis for esophageal varices - high-frequency knife: Hemostasis for esophageal varices is typically achieved through band ligation or sclerotherapy, not with a high-frequency knife.
  • c) Removal of foreign bodies in the stomach - clip: Removal of foreign bodies is usually performed with a grasping tool like forceps or a snare, not with clips, which are used for closing vessels or tissues.
  • d) Hemostasis for peptic ulcer - snare: Hemostasis for peptic ulcers is more commonly achieved using endoscopic injection, thermal coagulation, or clipping, rather than a snare, which is primarily used for polypectomy.
  • e) Submucosal dissection of early-stage gastric cancer - basket forceps: Endoscopic submucosal dissection (ESD) of early-stage gastric cancer is performed with a specialized knife, such as an IT knife or Hook knife, not basket forceps, which are used to remove polyps or other small objects.

13-10 (108G26) Which is true about nutrition management in the elderly?

a) Tube feeding infusion is carried out in the supine position.

b) Oral care is useful for the prevention of aspiration pneumonia.

c) Aspiration does not occur with tube feeding from a gastrostomy.

d) A nasogastric tube is less uncomfortable than tube feeding by gastrostomy.

e) Central venous nutrition has fewer infection complications compared to enteral nutrition.

 

The correct statement regarding nutrition management in the elderly is:

b) Oral care is beneficial for preventing aspiration pneumonia.

Explanation:

 

Good oral care is essential for maintaining oral hygiene and health, especially in the elderly, who may be at higher risk for aspiration pneumonia. Aspiration pneumonia can occur when food, saliva, liquids, or vomit is breathed into the lungs or airways leading to the lungs, which can be more common in those with difficulty swallowing or maintaining oral hygiene. Regular oral care helps reduce the amount of bacteria in the mouth, potentially decreasing the risk of aspiration and the development of pneumonia. This practice is particularly important for elderly patients who may rely on tube feeding or have compromised swallowing mechanisms.

13-11 (109B25) Which of the following statements is true about nutritional management for older adults who cannot take food orally?

a) Do not administer fluids through a nasogastric tube.

b) At the beginning of tube feeding, gradually increase the dose.

c) Oral care is not required during intravenous nutrition.

d) Do not use fatty preparations during intravenous nutrition therapy.

e) Swallowing function assessment will not be performed after the initiation of tube feeding.

 

The correct statement about nutritional management for older adults who cannot take it orally is:

b) At the beginning of tube feeding, gradually increase the dose.

Explanation:

 

For older adults who cannot take nutrition orally, beginning tube feeding with a gradual increase in the dose is essential to prevent complications such as aspiration pneumonia, diarrhea, or metabolic imbalances. This approach allows the digestive system to adjust to the feeding and can help prevent gastrointestinal discomfort. Additionally, gradually increasing the dose helps to identify the patient's tolerance level to the feeding, ensuring that the patient receives adequate nutrition without overwhelming the digestive system. The other options either introduce risks (such as ignoring the importance of oral care) or are incorrect practices (such as not administering fluids through a nasogastric tube or not assessing swallowing function after initiating tube feeding).

13-12 (109D10) Which of the following is true after percutaneous gastrostomy using a balloon-type gastrostomy catheter?

a) Change the catheter once a year.

b) Pull the catheter firmly to secure it to the abdominal wall.

c) Inject a 30% acetic acid solution into the catheter each time.

d) Inject saline into the balloon.

e) There is less discomfort during placement than with a nasogastric tube.

 

The correct statement regarding the care and management after percutaneous gastrostomy using a balloon-type gastrostomy catheter is:

e) There is less discomfort during placement than with a nasogastric tube.

Explanation:

  • a) Change the catheter once a year. This statement is partially correct; the frequency of changing a gastrostomy catheter can vary based on patient needs, catheter condition, and healthcare provider recommendations. However, it's not a universal rule to change it precisely once a year.

  • b) Pull the catheter firmly to secure it to the abdominal wall. This is incorrect because pulling the catheter firmly could cause trauma or dislodge it. The catheter should be securely placed but not pulled hard against the abdominal wall.

  • c) Inject a 30% acetic acid solution into the catheter each time. This is incorrect. Acetic acid is not typically used for catheter maintenance due to its potential for irritation and damage to the catheter or tissue.

  • d) Inject saline into the balloon. Specifically, distilled water is recommended for use in the balloon instead of tap water or saline to prevent coagulation and ensure that the water can be easily released.

  • e) There is less discomfort during placement than with a nasogastric tube. The nasogastric tube, which passes through the nose and throat, often causes significant discomfort due to irritation of the nasal passages and throat. In contrast, once a percutaneous gastrostomy is established and properly managed, the discomfort is relatively minor, making it a preferable option for long-term nutritional support in patients who require it.

13-13 (109D18) Which of the following statements are true regarding the perioperative management of radical resection for advanced mid-thoracic esophageal cancer? Select two.

a) Malnutrition is common before surgery.

b) Before surgery, solid foods are more easily consumed than liquids.

c) Oral care is beneficial for preventing postoperative pneumonia.

d) Early postoperative tube feeding is contraindicated.

e) Long-term central venous nutrition is administered postoperatively.

 

The correct statements in this context are:

a) There is a lot of malnutrition before surgery.

c) Oral care is useful for the prevention of postoperative pneumonia.

Explanation:

  • a) Malnutrition before surgery: Patients with advanced esophageal cancer frequently experience malnutrition before surgery. This is due to the tumor obstructing the esophagus, making it difficult for the patient to eat and absorb nutrients properly. Malnutrition can adversely affect the patient's overall condition, increasing the risk of complications and prolonging recovery time. Preoperative nutritional support is essential to improve the patient's nutritional status and support better postoperative outcomes.
  • c) Oral care for postoperative pneumonia prevention: Good oral hygiene and care before and after surgery can significantly reduce the risk of postoperative pneumonia. This is because oral care reduces the bacterial load in the mouth, decreasing the risk of aspiration of harmful pathogens into the lungs, which can cause pneumonia. Implementing a regimen of oral care in the perioperative phase is a straightforward yet effective measure to enhance patient outcomes.
  • b) Consumption of solid food before surgery: This statement is generally not true, as patients with mid-thoracic esophageal cancer often have difficulty swallowing solid foods due to the obstruction caused by the tumor.
  • d) Tube feeding in the early postoperative period: Tube feeding is not contraindicated in the early postoperative period; rather, it's often essential for providing nutrition when oral intake is not feasible or advisable.
  • e) Long-term central venous feeding postoperatively: While central venous feeding might be necessary in some cases, the goal is usually to resume enteral feeding as soon as possible because it is associated with fewer complications and promotes gut integrity and function. Long-term central venous feeding is typically reserved for cases where enteral feeding is not possible.

13-14 (109G28) Which combination of disease and endoscopic treatment is correct?

a) Small Bowel Vascular Dysplasia - Sclerotherapy

b) Gastric ulcer with exposed vessel - mucosal resection

c) Coloreorectal diverticular bleeding - clipping

d) Colorectal diverticulitis - localized therapy

e) Gastric varices - argon plasma coagulation

 

The most appropriate match between a disease and its endoscopic treatment among the given options is:

c) Colorectal diverticular bleeding - clipping

Explanation:

 

Endoscopic clipping is a common and effective treatment for controlling bleeding from colorectal diverticula. This technique allows for the direct application of clips to the bleeding source, providing a mechanical means to stop the bleed. It is especially useful in cases of acute diverticular bleeding, where locating the exact source of bleeding is possible through endoscopy. The other options do not traditionally match the listed treatments: sclerotherapy is more commonly used for esophageal varices rather than small bowel vascular dysplasia; mucosal resection is a method to remove lesions, such as early-stage cancers or polyps, rather than treat a bleeding gastric ulcer (though it could be indirectly related); localized therapy for colorectal diverticulitis is not typically an endoscopic treatment as diverticulitis often requires broader medical or surgical intervention; and argon plasma coagulation (APC) is indeed used for gastric varices, but it's more commonly associated with controlling bleeding from vascular lesions or in palliative care settings to reduce tumor bulk.

13-15 (111A12) Which combination of disease and symptom is incorrect?

a) Norovirus infection - vomiting

b) Hyperthyroidism - constipation

c) Upper gastrointestinal bleeding - black stool

d) Ulcerative colitis - mucous stool

e) Gastrointestinal obstruction - abdominal pain

 

The correct answer is:

b) Hyperthyroidism - constipation

Explanation:

 

Typically, hyperthyroidism, a condition characterized by an overactive thyroid gland producing an excess of thyroid hormones, leads to symptoms including weight loss, increased heart rate, anxiety, and more frequent bowel movements or diarrhea, not constipation. Constipation is more commonly associated with hypothyroidism, where the thyroid gland is underactive, slowing down many of the body's processes, including digestion.

13-16 (111B11) Which is used in interventional radiology (IVR) for duodenal ulcer bleeding when endoscopic hemostasis is challenging?

a) Ethanol

b) Clip

c) Coil

d) Stent

e) Filter

 

The correct answer is:

c) Coil

Explanation:

 

For duodenal ulcer bleeding where endoscopic hemostasis is difficult, Coil is often used in interventional radiology (IVR). Coil embolization is a technique that can directly target the bleeding vessel, allowing for effective cessation of bleeding. This method involves the insertion of a coil (or coils) into the bleeding artery to induce clotting and stop the bleed. Ethanol (a) and clips (b) are more commonly associated with endoscopic procedures rather than IVR. Stents (d) are typically used to maintain the patency of a vessel or lumen rather than directly stop bleeding. Filters (e) are used to prevent embolization to distant sites, such as in the case of preventing pulmonary embolisms, and are not used for hemorrhage control. Coil embolization's targeted approach and minimal invasiveness make it a valuable tool in cases where endoscopic methods are insufficient.

13-17 (112C3) Which are the less frequent symptoms of malabsorption syndrome?

a) Anemia

b) Edema

c) Constipation

d) Weight loss

e) Bloating

 

The correct answer is:

c) Constipation

Explanation:

  • The less frequent symptom of malabsorption syndrome among the options provided is Constipation. Malabsorption syndrome typically leads to symptoms related to the inability to absorb nutrients, vitamins, and minerals from the intestinal tract into the bloodstream, causing a variety of symptoms related to nutrient deficiencies and the passage of unabsorbed substances through the gut.
  • Anemia (a)can result from malabsorption of iron, vitamin B12, and folate.
  • Edema (b) might occur due to protein loss, particularly albumin, leading to low plasma oncotic pressure.
  • Weight loss (d) and bloating (e) are common because nutrients are not adequately absorbed, and unabsorbed food in the gut can ferment, causing gas and bloating.

13-18 (115A2) Which is the appropriate site for the placement of the drain tip for intraperitoneal drainage after sigmoid resection?

a) Douglas fossa

b) Morrison's pouch

c) Foramen of Winslow

d) Left subphrenic space

e) Right subphrenic space

 

The correct answer is:

a) Douglas fossa

Explanation:

 

The most appropriate site for the placement of the drain tip for intraperitoneal drainage after sigmoid resection is the Douglas fossa. The Douglas fossa, also known as the rectouterine pouch in females or rectovesical pouch in males, is the lowest point of the peritoneal cavity in the pelvis. After sigmoid resection, fluids such as blood or inflammatory exudates are likely to collect in the pelvis due to gravity. Placing the drain tip in the Douglas fossa allows for effective drainage of these fluids, reducing the risk of postoperative complications such as abscess formation. Morrison's pouch (b) and the subphrenic spaces (d, e) are located in the upper abdomen and are not relevant for drainage after sigmoid resection, which is performed in the lower abdomen. The Foramen of Winslow (c) is an anatomical opening, not a site for drainage.

Clinical

14-1 (112D37) A 56-year-old man was hospitalized after a small bowel resection. Four days ago, he was brought in by ambulance due to the sudden onset of colicky pain throughout his abdomen.

Which of these is not appropriate for this patient?

a) Fasting for 1 month

b) Introduction of home parenteral nutrition

c) Prevention of sarcopenia

d) Modification of the enteral nutrition composition

e) Alteration in the method of enteral nutrition administration

 

The correct answer is:

a) Douglas fossa

Explanation:

 

The least appropriate action for this patient is Fasting for 1 month. After a small bowel resection, it's important to initiate nutritional support to promote healing and maintain the patient's nutritional status. Fasting for an extended period, such as 1 month, could lead to further nutritional deficiencies, hinder recovery, and increase the risk of complications such as sarcopenia (muscle wasting). Options b through e are all measures aimed at ensuring the patient receives adequate nutrition and managing the risks associated with enteral nutrition, making them more appropriate in the postoperative management of a patient following a small bowel resection.